Friday, December 12, 2014

Causality and radioactive decay


At the Catholic blog Vox Nova, mathematics professor David Cruz-Uribe writes:

I… am currently working through the metaphysics of St. Thomas Aquinas as part of his proofs of the existence of God… [S]ome possibly naive counter-examples from quantum mechanics come to mind.  For instance, discussing the principle that nothing can change without being affected externally, I immediately thought of the spontaneous decay of atoms and even of particles (e.g., so-called proton decay).

This might be a very naive question: my knowledge of quantum mechanics is rusty and probably out of date, and I know much, much less about scholastic metaphysics.  So can any of our readers point me to some useful references on this specific topic? 

I’ve discussed this issue before, and one of Cruz-Uribe’s readers directs him to a blog post of mine in which I responded to a version of this sort of objection raised by physicist Robert Oerter.  Unfortunately, the combox discussion that ensues largely consists of a couple of Cruz-Uribe’s readers competing with each other to see who can emit the most squid ink (though Brandon Watson manfully tries to shine some light into the darkness).  One reader starts things out by writing:

Feser’s… argument seems to boil down to saying, “Just because we can’t find a cause for quantum phenomena doesn’t mean there isn’t one.” … Thing is, Bell has shown that you can’t have local unknown variables in quantum events. Bohm’s interpretation would give you the possibility of unknown variables (thus taking out the random, seemingly acausal, aspect), but at the price of locality (in short, such variables would be global, and not tied to a specific location; so you lose any predictability, anyway).

As readers of the post on Oerter know, this essentially just repeats the completely point-missing objection from Oerter that was the subject of the post, while ignoring what I said in the post in reply to the objection!  The combox discussion goes downhill from there, with so many points missed, questions begged and crucial distinctions blurred that you’d think you were reading Jerry Coyne’s blog. 

Cruz-Uribe’s reader accuses me of having a “weak” understanding of the relevant physics, which is why he launches into the mini lesson on Bell and Bohm.  But it’s his reading skills that are weak, since I made it clear in the post that I wasn’t in the first place making any claim about the physics of systems of the sort in question, and thus wasn’t saying anything that could be incompatible with what we know from physics.  In particular, I wasn’t advocating a “hidden variable theory” or the like, but rather making a purely philosophical point about causality that is entirely independent of such theories.

This is one of many factors that hinder fruitful discussion of these topics even with well-meaning people (like Cruz-Uribe) who know some science but know little philosophy.  They constantly translate philosophical claims into the physics terms that they feel more comfortable and familiar with, and proceed to run off at high speed in the wrong direction. 

This is why you really can’t address specific issues like radioactive decay without first doing some general philosophical stage-setting.  For it’s never really the empirical or scientific details that are doing the work in objections to Scholastic metaphysics like the one at issue.  What’s really doing the work is the ton of philosophical baggage that the critics unreflectively bring to bear on the subject -- the assumptions they read into the physics and then read back out again, thinking they’ve raised a “scientific” objection when what they’ve really done is raised a question-begging philosophical objection disguised as a scientific objection.  

(I imagine that educated religious people like Cruz-Uribe and his readers aren’t fooled by this kind of sleight of hand in other scientific contexts.  For instance, I’d wager that they would be unimpressed by arguments to the effect that neuroscience has shown that free will is an illusion.  As I have argued here and here, neuroscience has shown no such thing, and such claims invariably rest not on science but on tendentious philosophical assumptions that have been read into the scientific findings.  But exactly the same thing is true of claims to the effect that quantum mechanics has falsified the principle of causality, or that Newton or Einstein refuted the Aristotelian analysis of change.)

In what follows, then, I will first prepare the ground by calling attention to some common fallacies committed by critics of Scholastic metaphysics who appeal to modern physics -- fallacies some of which are committed by Cruz-Uribe’s readers in the course of their combox discussion.  Anyone wanting to comment intelligently on the subject at hand has to take care to avoid these fallacies.  Second, I will make some general remarks about what a philosophical approach to the subject at hand involves, as opposed to the approach taken by physics.  (I’ve discussed this issue many times before, and indeed did so in a couple of posts -- here and here -- that followed up the post on Oerter that Cruz-Uribe and his readers were discussing.)  Finally, in light of this background I’ll address the specific issue of radioactive decay and causality.

Fads and fallacies in the name of science

So, let’s consider some of the confusions that are rife in discussions of the relationship between physics on the one hand and philosophy (and in particular Scholastic philosophy) on the other:

A. Conflating empirical and metaphysical issues: Those who know some science but not a lot of philosophy very often assume that when a Scholastic philosopher says something about the nature of causality, or substance, or matter, or the like, then he is making a claim that stands or falls with what physics tells us, or at any rate should stand or fall with what physics tells us.  But this is a category mistake.  Scholastic metaphysics is not in competition with physics, but approaches the phenomena at a different (and indeed deeper) level of analysis.  Its claims do not stand or fall with the findings of physics, any more than the claims of arithmetic stand or fall with the findings of physics.  Indeed, like arithmetic, the basic theses of Scholastic metaphysics are (so the Scholastic argues) something any possible physics must presuppose.

Sometimes the critics assume that Scholastic metaphysics is in competition with physics because they are themselves making question-begging metaphysical assumptions.  For instance, they might assume that any rationally justifiable claim about the nature of matter simply must be susceptible of formulation in the mathematical language of physics, or must be susceptible of empirical falsification.  They are essentially making a metaphysics out of physics.  Only physics can tell us anything about the nature of physical reality (so the critic supposes), so any claim about the nature of physical reality is implicitly, even if not explicitly, a claim of physics.  As we will see below, this cannot possibly be right.  Physics cannot even in principle tell us everything there is to know about physical reality (let alone reality more generally).  But even if the assumption in question could be right, it simply begs the question against the Scholastic merely to assert it, since the Scholastic rejects this assumption, and on the basis of arguments that need to be answered rather than ignored (arguments I’ll discuss below). 

Sometimes the conflation of empirical and metaphysical issues is due less to such large-scale philosophical assumptions than to a simple fallacy of equivocation.  Both physicists and Scholastic metaphysicians use terms like “cause,” “matter,” and the like.  A superficial reading therefore often leads critics to assume that they are addressing the same issues, when in fact they are very often not using the key terms in the same sense. 

Sometimes the conflation is due to sheer intellectual sloppiness.  Critics will formulate the issues in ridiculously sweeping terms, making peremptory claims to the effect that “Aristotelianism was refuted by modern science,” for example.  In fact, of course, the labels “Aristotelianism” and “modern science” each cover a large number of distinct and logically independent ideas and arguments, and these need carefully to be disentangled before the question of the relationship between Scholastic metaphysics and modern physics can fruitfully be addressed.  It is no good to say (for example) that since Aristotle’s geocentrism and theory of natural place have been falsified, “therefore” we should not take seriously his theory of act and potency or the account of causality that rests on it.  This is simply a non sequitur.  Such issues are completely independent of one another, logically speaking (regardless of the contingent historical association between them).

B. Conflating genus and species: Even when physicists and Scholastic metaphysicians are using terms in the same sense, critics often confuse what is really only a specific instance of the general class named by a term with the general class itself.  For example, where the notion of “cause” is concerned, Scholastic metaphysicians distinguish between formal, material, efficient, and final causes.  Where efficient causes are concerned, they distinguish between principal and instrumental causes, between series of causes which are essentially ordered and those which are accidentally ordered, and between those which operate simultaneously versus those which are ordered in time.  They distinguish between total causes and partial causes, and between proximate and remote causes.  They regard causality as primarily a feature of substances and only secondarily as a relation between events.  They distinguish between causal powers and the operation of those powers, between active causal power and passive potencies.  And so forth.  All of these distinctions are backed by arguments, and the Scholastic maintains that they are all necessary in order to capture the complexity of causal relations as they exist in the actual world. 

Now, those who criticize Scholastic metaphysics on scientific grounds typically operate with a very narrow understanding of causality.  In particular, they often conceive of it as a deterministic relation holding between temporally separated events.  They will then argue (for example) that quantum mechanics has undermined causality thus understood, and conclude that it has therefore undermined causality full stop.  One problem with this, of course, is that whether quantum mechanics really is incompatible with determinism is a matter of controversy, though as I have said, nothing in the Scholastic position stands or falls with the defensibility of Bohmian hidden variable theories.  The deeper point is that it is simply fallacious to suppose that to undermine one kind of causality (and in one kind of context) is to undermine causality as such.  Certainly it begs the question against the Scholastic, who denies that all causality reduces to deterministic relations holding between temporally separated events.

The conflation of a general class with a specific kind within the general class is evident too in discussions of motion.  Scholastics and other Aristotelians think of motion in general as change, and change as the actualization of potency.  Local motion or change with respect to place or location is just one kind of actualization of a potency, and is metaphysically less fundamental than other kinds.  When motion is discussed in modern physics, however, it is of course local motion that is exclusively in view. 

There is nothing necessarily wrong with this focus, but it would be fallacious to draw, from what modern physics says about “motion” (in the sense of local motion), sweeping conclusions about what Aristotelians say about “motion” (in the sense of the actualization of potency).  This would be to confuse what is true of one kind of change for what is true of change as such.  Yet this kind of fallacious conflation is very common.  Of course, a critic of Scholastic metaphysics might claim that local motion is the only kind of change there really is, but merely to assert this is simply to beg the question against the Scholastic, who has arguments for the claim that local motion cannot be the only kind of change there is.  (I have addressed this particular issue in detail elsewhere, e.g. here and here.)

C. Confusing general principles with specific applications of those principles: When a thinker, whether a philosopher or a scientist, puts forward a general principle, he sometimes illustrates it with examples that later turn out to be deficient.  But it simply doesn’t follow that the general principle itself is mistaken.  For example, people often think of the evolution of the horse as a neat transition from very small animals to ever larger ones, as in the kind of exhibit they might have seen in a natural history museum as a child.  It turns out that things aren’t quite so neat.  There is no hard and fast correlation between the size of a horse and where it appears in the fossil record.  It doesn’t follow, however, that modern horses did not evolve from much smaller animals.  That earlier accounts of the evolution of the horse turn out to be mistaken does not entail that the general principle that horses evolved is mistaken.  (ID enthusiasts are kindly asked to spare us any frantic comments about evolution.  This is not a post about that subject.  It’s just an example.) 

However, though philosophical naturalists never tire of making this point when Darwinism is in question, they suddenly forget it when Aristotelianism or Scholasticism is what is at issue.  For example, Aristotelians defend the reality of final causality -- the idea that natural substances and processes are inherently “directed towards” certain characteristic effects or ranges of effects.  In previous centuries, the idea was often illustrated in terms of Aristotle’s view that heavy objects are naturally directed toward the center of the earth as their “natural place.”  That turns out to be mistaken.  This is often treated as a reason for rejecting the idea of final causality as such, but this simply doesn’t follow.  In general, the deficiencies of this or that illustration of some Scholastic metaphysical thesis are simply not grounds for rejecting the thesis itself.  (I’ve addressed this issue at greater length before, e.g. here, here, and here.)

The limits of physics

So that’s one set of background considerations that must be kept in mind when addressing topics like the one at issue: the begged questions, blurred distinctions, and missed points which  chronically afflict the thinking of those who raise purportedly scientific objections to Scholastic metaphysics.  Let’s move on now to the second set of background considerations, viz. the limits in principle to what physics can tell us about physical reality, and the unavoidability of a deeper metaphysical perspective. 

As I have emphasized many times, what physics gives us is a description of the mathematical structure of physical reality.  It abstracts from any aspect of reality which cannot be captured via its exclusively quantitative methods.  One reason that this is crucial to keep in mind is that from the fact that something doesn’t show up in the description physics gives us, it doesn’t follow that it isn’t there in the physical world.  This is like concluding from the fact that color doesn’t show up in a black and white pen and ink drawing of a banana that bananas must not really be yellow.  It both cases the absence is an artifact of the method employed, and has nothing whatsoever to do with the reality the method is being used to represent.  The method of representing an object using black ink on white paper will necessarily leave out color even if it is there, and the method of representing physical reality using exclusively mathematical language will necessarily leave out any aspect of physical reality which is not reducible to the quantitative, even if such aspects are there.

But it’s not just that such aspects might be there.  They must be there.  The quantitative description physics gives us is essentially a description of mathematical structure.  But mathematical structure by itself is a mere abstraction.  It cannot be all there is, because structure presupposes something concrete which has the structure.  Indeed, physics itself tells us that the abstraction cannot be all there is, since it tells us that some abstract mathematical structures do not fit the actual, concrete material world.  For example, Einstein is commonly taken to have shown that our world is not really Euclidean.  This could only be true if there is some concrete reality that instantiates a non-Euclidean abstract structure rather than a Euclidean abstract structure.  So, physics itself implies that there must be more to the world than the abstract structure it captures in its purely mathematical description, but it does not and cannot tell us exactly what this concrete reality is like. 

That physics by itself only gives us abstract structure is by no means either a new point or a point emphasized by Scholastics alone.  It was made in earlier generations by thinkers like Poincaré, Russell, Eddington, Weyl, and others, and in recent philosophy has been emphasized by Grover Maxwell, Michael Lockwood, Simon Blackburn, David Chalmers, and others. 

Moreover, we know there must be more to causality specifically than physics does or could tell us about.  The early Russell once argued that causation must not be a real feature of the world precisely because it does not show up in the description of the world physics gives us.  For physics, says Russell, describes the world in terms of differential equations describing functional relations between events, and these equations make no reference to causes.  “In the motions of mutually gravitating bodies, there is nothing that can be called a cause, and nothing that can be called an effect; there is merely a formula” (“On the Notion of Cause,” pp. 173-74).  Russell’s position has been the subject of a fair bit of attention in recent philosophy (e.g. here). 

Now, I don’t myself think it is quite right to say that physics makes no use of causal notions, since I think that physics tells us something about the dispositional features of fundamental particles, and dispositionality is a causal notion.  Still, as other philosophers have argued, higher-level causal features -- such as the causation we take ourselves to experience continuously in everyday life, in the behavior of tables, chairs, rocks, trees, and other ordinary objects -- are more difficult to cash out in terms of what is going on at the micro level described by physics.  Hilary Putnam is one contemporary philosopher who has addressed this problem, as I noted in a post from a few years ago.  Trenton Merricks is another, and argues that at least macro-level inanimate objects are unreal, since (he claims) they play no causal role in the world over and above the causal role played by their microphysical parts.

Merricks thinks living things are real, and certainly a Russell-style across-the-board denial of causation would be incoherent, for a reason implicit in a fact that the later Russell himself emphasized.  Our perceptual experiences give us knowledge of the external physical world only because they are causally related to that world.  To deny causality in the name of science would therefore be to undermine the very empirical foundations of science. 

Now, if there must be causality at the macro level (at the very least in the case of the causal relations between the external world and our perceptual experiences of it), and this causality is not captured in the description of the world that physics itself gives us, then it follows that there is more to causality than physics can tell us.  And even if you dispute the views of Russell, Putnam, Merricks, et al., physics itself is not going to settle the matter.  For it is not an empirical matter, but a philosophical dispute about how to interpret the empirical evidence.

(Nor will it do to dismiss such disputes on the grounds that the competing views about them are “unfalsifiable.”  It may be that there is no human being more comically clueless than the New Atheist combox troll who thinks he can dismiss philosophy on grounds of falsificationism -- a thesis put forward by a philosopher, Karl Popper.  As Popper himself realized, falsificationism is not itself a scientific thesis but a meta-level claim about science.)

If physics in general raises philosophical questions it cannot answer, the same is if anything even more clearly true of quantum mechanics in particular.  Feynman’s famous remark that nobody understands quantum mechanics is an overstatement, but it is certainly by no means obvious how to interpret some of the theory’s stranger aspects.  Quantum mechanics has been claimed to “show” all sorts of things -- that the law of excluded middle is false, that scientific realism is false, that idealism is true, etc.  By itself it shows none of these things.  In each case, certain philosophical assumptions are first read into quantum mechanics and then read out again.  But the same thing is true of claims to the effect that quantum mechanics undermines causality.  By itself it does not, and could not, show such a thing either.  Here as in the other cases, it is the metaphysical background assumptions we bring to bear on quantum mechanics that determine how we interpret it.  This is as true of philosophical naturalists, atheists, et al. as it is of Scholastics. 

Now, the Scholastic metaphysician argues, on grounds entirely independent of questions about how to interpret quantum mechanics, that there are a number of metaphysical theses that any possible empirical science is going to have to presuppose.  Most fundamentally, there is the Aristotelian theory of act and potency, according to which we cannot make sense of change as a real feature of the world unless we recognize that there is, in addition to what is actual on the one hand, and sheer nothingness on the other, a middle ground of potentiality.  Change is the actualization of a potentiality, and unless we affirm this we will be stuck with a static Parmenidean conception of the world.  And that is not an option, because the existence of change cannot coherently be denied.  Even to work through the steps of an argument for the non-existence of change is itself an instance of change.  Sensory experience – and thus the observation and experiment on which empirical science rests – presupposes real change.  (Hence it is incoherent to suggest, as is sometimes done, that relativity shows that change is illusory, since the evidence for relativity presupposes sensory experience and thus change.)

Now, the main concepts of the Aristotelian-Scholastic metaphysical apparatus – substantial form and prime matter, final causality and efficient causality, and so forth – are essentially an outworking of the theory of act and potency.  You can argue about whether this or that object truly has a substantial form or is merely an aggregate, about whether we have correctly identified and characterized the teleological features of such-and-such a natural process, and so on.  What cannot be denied is that substantial form, teleology, etc. are bedrock features of the natural order and will inevitably feature in a complete picture of the physical world at some level of analysis.  All of that follows from a consistent application of the theory of act and potency.  It also cannot be denied that any potential that is actualized is actualized by something already actual.  That is the core of the “principle of causality,” and It follows from the principle of sufficient reason -- a principle which, rightly understood, also cannot coherently be denied.  

I spell out the reasons for all of this in detail, and also discuss the inherent limitations of empirical science, in Scholastic Metaphysics.  The point to emphasize for present purposes is that the Scholastic holds that there a number of general metaphysical truths which we can know completely independently of particular disputes within physics or any other empirical science, precisely because they rest on what any possible empirical science must itself presuppose.  (One of Cruz-Uribe’s readers insinuates that in resting its key theses on something other than empirical science, Scholastic metaphysics undermines the possibility of any common ground with its critics.  But this is precisely the reverse of the truth and once again completely misses the point.  Since Scholastic metaphysical arguments begin with what empirical science presupposes -- for example, the possibility of sensory experience, and the possibility of at least partial explanations -- they thereby begin precisely with what the critics already accept, not with what they reject.)

Radioactive decay

So, here is where we are before we even get to the issue of radioactive decay:  Purportedly physics-based objections to Scholastic metaphysics – including objections to Scholastic claims about causality -- are, as a matter of course, poorly thought out.  They commonly blur the distinction between empirical and philosophical claims, confuse what is really only one notion of causality with causality as such, and confuse mere illustrations or applications of general metaphysical principles with the principles themselves.  Meanwhile, we know on independent grounds that physics, of its very nature, cannot in principle tell us everything there is to know about physical reality, including especially the causal features of physical reality.  Its exclusively mathematical conceptual apparatus necessarily leaves out whatever cannot be captured in quantitative terms.  Physics also implies that there must be something more to physical reality than what it captures, since mathematical structure is of itself a mere abstraction and there must be some concrete reality which has the structure.

We also know that quantum mechanics in particular raises all sorts of puzzling metaphysical questions (not merely about causality) that it cannot answer.  And, the Scholastic argues, we know on independent grounds – grounds that any possible empirical science must presuppose – that there are a number of metaphysical truths that we must bring to bear on our understanding of the world whatever the specific empirical facts turn out to be, including the truth that causality must be a real feature of the world.

So, when critics glibly allege that radioactive decay or other quantum phenomena undermine causality, the trouble is that they are making a charge that doesn’t even rise to the level of being well thought out.  It is preposterous to pretend that the burden of proof is on the Scholastic to show that quantum mechanics is compatible with Scholastic claims about causality.  The burden of proof is rather on the critic to show that there really is any incompatibility.  (Few people would claim that the burden of proof is on anyone to prove that quantum mechanics doesn’t establish idealism, or doesn’t undermine the law of excluded middle, or doesn’t refute scientific realism.  It is generally realized that the claims in question here are very large ones that go well beyond anything quantum mechanics itself can be said to establish, so that the burden of proof is on anyone who wants to claim quantum mechanics has such sweeping implications.  So why is the burden of proof on the Scholastic to show that quantum mechanics doesn’t undermine causality?)

In particular, the critic owes us an account of why, since physics cannot in principle capture all there is to physical reality in the first place -- and in particular arguably fails entirely (as Russell held) to capture causality in general -- we should regard it as especially noteworthy if it fails to capture causality in one particular case.  If the critic, like the early Russell, denies that there is any causality at all, he owes us an account of how he can coherently take such a position, and in particular how he can account for our knowledge of the world physics tells us about if we have no causal contact with it.  If the critic says instead that genuine causality does exist in some parts of nature but not in the particular cases he thinks quantum mechanics casts doubt on, he owes us an account of why we should draw the line where he says we should, and how there could be such a line.  (As we had reason to note recently with respect to PSR, it is difficult to see how it could be coherent to think that things are in principle explicable in some cases while denying that they are in general explicable in principle.  Yet to affirm the principle of causality in some cases and deny it in others seems similarly incoherent.) 

In short, anyone who claims that quantum mechanics undermines Scholastic metaphysical claims about causality owes us an alternative worked-out metaphysical picture before we should take him seriously (just as anyone who would claim that quantum mechanics undermines the law of excluded middle owes us an alternative system of logic if we are to take him seriously).  And if he gives us one, it would really be that metaphysical system itself, rather than quantum mechanics per se, that is doing the heavy lifting.

Now, no one expects a logician to launch into a mini treatise on quantum mechanics before setting forth a textbook exposition of classical logic, law of excluded middle and all.  The reason is that it is widely understood that it is just false to say flatly that “Quantum mechanics has undermined classical logic.”  Quantum mechanics has done no such thing.  Rather, some people have been led by their metaphysical speculations about quantum mechanics to wonder whether logic might be rewritten without the law of excluded middle.  Logicians who have independent grounds to think that the law of excluded middle cannot be false have no reason to take these speculations very seriously or respond in detail to them when going about their ordinary work.

Similarly, there is no reason why a Scholastic metaphysician should be expected to launch into a detailed discussion of quantum mechanics before deploying the principle of causality in a general metaphysical context, or when giving an argument for the existence of God.  For it is also simply false to say that “Quantum mechanics has undermined the principle of causality.”  It has done no such thing.  The most that one can say is that some people have been led by their metaphysical speculations about quantum mechanics to wonder whether metaphysics might be rewritten in a way that does without the principle of causality.  But metaphysicians who have independent grounds to think that the principle of causality cannot be false have no reason to take these speculations very seriously or to respond in detail to them when going about their ordinary work.

Of course, logicians have examined proposed non-classical systems of logic, and classical logicians have put forward criticisms of these alternative systems.  The point is that their doing so is not a prerequisite of their being rationally justified in using classical logic.  Similarly, a Scholastic metaphysician, especially if he is interested in questions about philosophy of nature and philosophy of physics, can and should address questions about how to interpret various puzzling aspects of quantum mechanics.  But the point is that doing so is not a prerequisite to his being rationally justified in appealing to the principle of causality in general metaphysics or in presenting a First Cause argument for the existence of God.

But how might a Scholastic interpret phenomena like radioactive decay?  I hinted at one possible approach in the post on Oerter linked to above, an approach which is suggested by the way some Scholastic philosophers have thought about local motion.  Some of these thinkers, and Aquinas in particular, take the view that a substance can manifest certain dispositions in a “spontaneous” way in the sense that these manifestations simply follow from its nature or substantial form.  A thing’s natural tendencies vis-à-vis local motion would be an example.  These motions simply follow from the thing’s substantial form and do not require a continuously conjoined external mover.  Now, that is not to say that the motion in question does not have an efficient cause.  But the efficient cause is just whatever generated the substance and thus gave it the substantial form that accounts (qua formal cause) for its natural local motion.  (It is commonly but erroneously thought that medieval Aristotelians in general thought that all local motion as such required a continuously conjoined cause.  In fact that was true only of some of these thinkers, not all of them.  For detailed discussion of this issue, see James Weisheipl’s book Nature and Motion in the Middle Ages, from which I borrow the language of “spontaneity.”  I also discuss these issues in more detail here.)

Now, Aquinas himself elaborated on this idea in conjunction with the thesis that the “natural place” toward which heavy objects are inclined to move is the center of the earth, and he supposed also that projectile motions required a conjoined mover insofar as he regarded them as “violent” rather than natural.  Both of these suppositions are outmoded, but the more general thesis summarized in the preceding paragraph is logically independent of them and can easily be disentangled from them.  One can consistently affirm (a) that a substance will tend toward a certain kind of local motion simply because of its substantial form, while rejecting the claim that (b) this local motion involves movement toward a certain specific place, such as the center of the earth.  (This is a point missed by one of the more clueless commentators in Cruz-Uribe’s combox, whose capacity for grasping obvious distinctions is not much better than his reading ability.  He ridicules the distinction I make here without offering the slightest explanation of what exactly is wrong with it.) 

Indeed, some contemporary Aristotelians have proposed that affirming (a) while rejecting (b) is the right way to think about inertial motion: Newton’s principle of inertia, on this view, is a description of the way a physical object will tend to behave vis-à-vis local motion given its nature or substantial form.  (Again, see this article for discussion of the relevant literature.)  The point for present purposes, though, is that the idea just described also provides a model -- I don’t say it is the only model, just a model -- for understanding what is going on metaphysically with phenomena like radioactive decay. 

The idea would be this.  Let’s borrow an example from philosopher of science Phil Dowe’s book Physical Causation, since I’ll have reason to return to the use he makes of it in a moment.  Dowe writes:

Suppose that we have an unstable lead atom, say Pb210.  Such an atom may decay, without outside interference, by α-decay into the mercury atom Hg206.  Suppose the probability that the atom will decay in the next minute is x.  Then

                        P(E|C) = x

where C is the existence of the lead atom at a certain time t1, and E is the production of the mercury atom within the minute immediately following t1.  (pp. 22-23)

Now, applying the conceptual apparatus borrowed from Aquinas (which, I should add, Dowe himself does not do), we can say that the decay in question is “spontaneous” in something like the way Aquinas thought the natural local motion of a physical substance is “spontaneous.”  In particular, given the nature or substantial form of Pb210, there is a probability of x that it will decay in the next minute.  The probability is not unintelligible, but grounded in what it is to be Pb210 .  The decay thus has a cause in the sense that (i) it has a formal cause in the nature or substantial form of the particular Pb210 atom, and (ii) it has an efficient cause in whatever it was that originally generated that Pb210 atom (whenever that was). 

It is worth noting that you don’t need to be a Scholastic to think that there really is causation in cases like this, which brings me to Dowe’s own use of this example.  As Dowe notes, even if it is claimed that decay phenomena are incompatible with deterministic causality, it doesn’t follow that there is no causality at all in such cases.  All that would follow is that the causality is not deterministic.  In defense of the claim that there is causality of at least an indeterministic sort in cases like the one he cites, he writes:

If I bring a bucket of Pb210 into the room, and you get radiation sickness, then doubtless I am responsible for your ailment.  But in this type of case, I cannot be morally responsible for an action for which I am not causally responsible.  Now the causal chain linking my action and your sickness involves a connection constituted by numerous connections like the one just described.  Thus the insistence that C does not cause E on the grounds that there’s no deterministic link entails that I am not morally responsible for your sickness.  Which is sick.  (p. 23)

Dowe also points out that “scientists describe such cases of decay as instances of production of Hg206… [and] ‘production’ is a near-synonym for ‘causation’” (p. 23).  This sounds paradoxical only if we fallaciously conflate deterministic causality and causality as such.

Interestingly, elsewhere in his book, Dowe argues that Newton’s first law should be interpreted as entailing, not that a body’s uniform motion has no cause, but rather that its inertia, conceived of as a property of a body, is its cause (pp. 53-54).  This dovetails with the analysis of inertial motion given by some contemporary Aristotelians, to which I alluded above.  John Losee, in his book Theories of Causality, discusses Dowe’s views and notes the parallel between what Dowe says about radioactive decay and what he says about inertia (p. 126).  The parallel, I would say (using notions neither Dowe nor Losee appeal to), is this: In both cases, Dowe is describing the way a thing will “spontaneously” tend to behave given its nature or substantial form (albeit the manifestation of the tendency is probabilistic in the case of Pb210 but not in the case of inertial motion). 

So, Dowe’s views seem to some extent to recapitulate the elements of the Aquinas-inspired account of radioactive decay sketched above, which I earlier put forward in the post replying to Oerter.  It is worth emphasizing that neither Dowe nor Losee has any Scholastic ax to grind, and that I came across their work long after writing that post -- so as to forestall any objection to the effect that the proposed account is somehow a merely ad hoc way to try to get round the objection from radioactive decay (an objection that would be absurd in any case given that the basic concepts made use of in the proposed account are centuries old).  On the contrary, it is an account that someone could accept whatever his views about Scholastic metaphysics in general, or about the application of the principle of causality to arguments for God’s existence.

In any event, as I have said, the burden of proof is not on the Scholastic metaphysician to provide an account of how radioactive decay can be reconciled with the principle of causality, because claims to the effect that there is an incompatibility are not even well-motivated in the first place.  The burden of proof is rather on the critic of Scholastic metaphysics to develop an alternative metaphysical framework on which the rejection of the principle of causality is defensible, and within which the critic might embed his favored interpretation of quantum mechanics.  But don’t hold your breath.  For the Scholastic has grounds entirely independent of issues about quantum mechanics or radioactive decay to conclude that no such alternative metaphysics is forthcoming. 

483 comments:

  1. ... between series of causes which are essentially ordered and those which are accidentally ordered, and between those which operate simultaneously versus those which are ordered in time.


    This is incoherent.

    If causes operate simultaneously, then they are not causing each other and are not operating in series - by definition.

    Put another way, in a group of events occurring simultaneously - no member of the group can be, non-arbitrarily, referred to as a cause, or as an effect of any other member of said group.

    ReplyDelete

  2. Put another way, in a group of events

    And there's your problem.

    ReplyDelete
  3. Cruz-Uribe’s reader accuses me of having a “weak” understanding of the relevant physics, which is why he launches into the mini lesson on Bell and Bohm

    I recently found your blog through links to some of your posts in the past discussing certain evolutionary biology issues. I am not an expert on quantum mechanics, but I do claim to know my biology, and excuse me for saying it so openly, but you have made such gigantic blunders in your discussions of evolutionary topics, revealing glaring misunderstandings of basic concepts, that nobody who knows something about that subject could take your opinion seriously about anything biological. Is there any reason to think it is anything different with physics? I don't see much evidence to support that view. In the light of that, such lecturing is perfectly justified.

    Regarding the general topic. Your entire corpus of writings seems to be one big exercise in pointing out how philosophy has conclusively shown that modern science cannot derive any knowledge with absolute certainty. Well, duh. Tell me something I did not already know. Science is Bayesian in nature, and scientists understand that very well, and when they say that we "know" something, they do not mean that we know it with 100% certainty, just that is would be silly to think otherwise given the cumulative evidence available.

    The problem is that after you have written hundreds of thousands of words repeating that same point, you go to church to pray to your (apparently according to you demonstrated to exist with absolute certainty, with the very precise characteristics described in certain holy books) deity to save your (again, according to you, demonstrated to exist with absolute certainty) immaterial soul.

    Do you see the immense hypocrisy here?

    ReplyDelete
  4. The typos are strong with me tonight

    Bob,

    When I lay me down in a few moments here, my head will be supported by a pillow which will itself be caused to flatten a bit by my rather heavy skull.

    ReplyDelete
  5. @Georgi Marinov

    What are these "huge blunders" Feser makes?

    In your post I see nothing but pointless rethoric.

    Moreover Feser has always supported Evolution AGAINST

    Supporters of Intelligent Design and the like (in this post as well, btw).

    ---

    "Well, duh. Tell me something I did not already know. Science is Bayesian in nature, and scientists understand that very well, and when they say that we "know" something, they do not mean that we know it with 100% certainty, just that is would be silly to think otherwise given the cumulative evidence available. "

    Well I guess you are not that well informed, since many people, some scientists included, think science is the answer for everything.

    This "scientism" is not a small minority either.


    ---


    "The problem is that after you have written hundreds of thousands of words repeating that same point, you go to church to pray to your (apparently according to you demonstrated to exist with absolute certainty, with the very precise characteristics described in certain holy books) deity to save your (again, according to you, demonstrated to exist with absolute certainty) immaterial soul."


    So what if he does that?

    The only hypocricy I see here is yours... and a lot of intellectual dishonesty.

    ReplyDelete
  6. Summarizing Gergi Marinov's post we have:

    (1) Prof. Feser makes "gigantic blunders" in discussion of Evolution theory but we are offered no examples.

    (2) Prof. Feser's entire corpus is "one big exercise in pointing out how philosophy has conclusively shown that modern science cannot derive any knowledge with absolute certainty".

    (3) But the *real* problem is that Prof. Feser goes to Church, which marks him a Big Hypocrite.

    In truth I tell you, the trolls, emerging from disease ridden swamps, have arrived.

    ReplyDelete
  7. @Georgi Marinov

    ""Well, duh. Tell me something I did not already know. Science is Bayesian in nature, and scientists understand that very well, and when they say that we "know" something, they do not mean that we know it with 100% certainty, just that is would be silly to think otherwise given the cumulative evidence available. ""


    More about this.

    Feser (and many other philosophers as well!) critique is NOT that physics and empirical science are 'Bayesian in nature' (althought that is obvious) but rather that science CANNOT, even in principle, be the ONLY method to acheiving the whole truth about the universe.

    That's something very different.

    Take people like Dawkins or Krauss (who are your heroes probably, since they fail to read what other people write properly as well): they claim that physics and biology have all the answers on everything (or nearly so).

    Some scientist have also called their BS and so have several philosophers of science.

    Still Krauss and Dawkins idea is quite pernicious... and the problem of their idea has NOTHING to do with Bayesian nature of empirical science at all.

    =====


    ReplyDelete
  8. Ismael said...

    What are these "huge blunders" Feser makes?


    Let's start here:

    http://edwardfeser.blogspot.com/2011/09/modern-biology-and-original-sin-part-i.html

    Where he shows complete absence of udnerstanding of the concept of effective population size

    Moreover Feser has always supported Evolution AGAINST

    I see nothing of the sort. Evolutionary theory is nonteleological. If you are arguing that it has a direction determined by some deity, you are rejecting it. You may be accepting the fact of physical evolution, but the not theory. That's not a science-friendly attitude. The theory matters a lot.

    So what if he does that?

    In the real world, as opposed to the abstract one that philosophers like to spend way too much time in, we weigh ideas by the evidence that supports them. If you write at such length attacking scientists on the grounds of how science cannot prove things with absolute certainty, then unconditionally embrace ideas, which are many orders of magnitude less well supported, you cannot expect reasonable people to view you positively.

    ReplyDelete
  9. @Matt Sheean

    When I lay me down in a few moments here, my head will be supported by a pillow which will itself be caused to flatten a bit by my rather heavy skull.

    Sure, and this is relevant to my comment how?

    ReplyDelete
  10. <>Well I guess you are not that well informed, since many people, some scientists included, think science is the answer for everything.

    This "scientism" is not a small minority either.

    ....

    Take people like Dawkins or Krauss (who are your heroes probably, since they fail to read what other people write properly as well): they claim that physics and biology have all the answers on everything (or nearly so).


    1) I don't have "heroes", there are people I respect for their positive contributions to the advancement of knowledge, and other who I don't, for their negative such contributions

    2) You can find numerous instances on which each of the most popular "apostles of scientism" clearly says the same thing I said above.

    3) Scientism as a good thing

    ReplyDelete
  11. Ismael said...

    Feser (and many other philosophers as well!) critique is NOT that physics and empirical science are 'Bayesian in nature' (althought that is obvious) but rather that science CANNOT, even in principle, be the ONLY method to acheiving the whole truth about the universe.


    I know that argument well too. It's BS - you need to show that there are things that are inaccessible to science rather than merely assert their existence. Nobody has ever done such a thing.

    ReplyDelete
  12. RADIOACTIVE DECAY

    Now I always found quite STUPID when atheists try to use the "radioactive decay argument".

    First of all radioactive decay has a cause... actually at least 3 ;)


    1- A Material cause (obviously), since there would be no decay if there was not a material atom that decays.


    2- A Formal cause, since it is in the "nature" of atom X to decay, hence the "form" (in philosophical sense, http://www.britannica.com/EBchecked/topic/213675/form)


    After all, all atoms are made up by quarks and electrons, so the building blocks of all atoms are the same, still some are arranged differently than other, so they have a different form and different properties we can observe.

    3 - A Final cause (since they are "directed" albeit NOT consciously towards that end). I think the "final causation" is reflected in the laws of nature themselves, which indeed "direct" various objects to do what they do (eg gravity directs an apple to fall on the ground and potentially on Newton's head).

    ----


    Now:
    The efficient cause might not be apparent (although there are several theories, including that vacuum quantum fluctuations aid the process) but one cannot exclude it a priori either.

    In the end radioactive decay MUST come from SOME interaction between the quarks that make up the nucleus (and maybe something external like vacuum quantum fluctuations, but not necessarily).

    In a sense the "radioactive decay" is no more problematic to a Scholastic than "animal motion".

    Animals move "by themselves" without necessarily and apparent "external cause"...

    The "efficient cause" of an object is equivalent to that which causes change and motion to start or stop... but such efficient cause does not need to be necessarily external to the object itself, I would say.

    To not that a radioactive atom might exist forever without decaying, but has NOT existed forever in the past, i.e. it comes from something else (quarks and electrons coming together, let's say in very basic terms)

    ----

    Moreover:
    It is NOT very scientific to claim "oh it just happens for no reason at all".

    The REASON of science is to find out HOW things work... so to say "radioactive decay just happens" and leave it at that, simply betrays the scientific mind!


    That is why I think it is pathetic that atheists, who seem to worship science and think they are so big in it, would even consider making such arguments.

    ReplyDelete
  13. @ Georgi Marinov

    I think you ought to re-read what Feser wrote.


    "http://edwardfeser.blogspot.com/2011/09/modern-biology-and-original-sin-part-i.html

    Where he shows complete absence of udnerstanding of the concept of effective population size"


    As I recall he does nto per se argue agains the idea of a minimal population... yet even if he did make a mistake there, so what?

    Some Nobel prize winners have said some nonsense as well, should we invalidate then all their work?

    ---

    Moreover Feser has always supported Evolution AGAINST

    I see nothing of the sort. Evolutionary theory is nonteleological. If you are arguing that it has a direction determined by some deity, you are rejecting it. You may be accepting the fact of physical evolution, but the not theory. That's not a science-friendly attitude. The theory matters a lot.


    Thay you really have NOT been reading properly.

    Intelligence Design is not the same a "teleology", certainly not in a scholastic sense.

    First of all you say:
    "Evolutionary theory is nonteleological. If you are arguing that it has a direction determined by some deity, you are rejecting it."

    Sure evolutionary theory rejects A PRIORI some teleology, but this does NOT mean the teleology is not there.

    HOWEVER (and you should really read Feser again) this does NOT mean Gos is "directing evolution" by tweaking and intervening constantly.

    That would be intelligence design.

    ON THE OTHER HAND, the very fact that objects FOLLOW THE LAWS OF PHYSICS is teleology.

    An electron repelling another electron by coulomb interaction, for example. Electrons are "directed" at repelling each other.

    (again this must not be read in the intelligent design sense, where God tells the electrons to do so or something and does so for each electron)

    ---


    " You may be accepting the fact of physical evolution, but the not theory. That's not a science-friendly attitude. The theory matters a lot. "

    You are greatly CONFUSED.
    This is the main problem with people today.

    The theory opf evolution (which many SCIENTISTS thinks needs huge revisions: http://www.nature.com/news/does-evolutionary-theory-need-a-rethink-1.16080 a Nature article, note!) is simply a "model for reality".

    It might "disregard" teleology, but even so it does NOT deny it either.

    Those who DO deny it (like you) do it through a PHILOSOPHICAL interpretation of the theory of evolution (much like those who have various interpretations for quantum mechanics).

    Such interpretations are NOT science, however, but philosphy.

    People like you might THINK they are "science"... but then one should worry that self-proclaimed experts like you do not even understand the difference between a "basic theory" and "interpretations" of such theory.

    ReplyDelete
  14. @Ed

    That was a monster of a blog post. How on earth do you do it? I mean with a job, kids and all.

    ReplyDelete
  15. Feser (and many other philosophers as well!) critique is NOT that physics and empirical science are 'Bayesian in nature' (althought that is obvious) but rather that science CANNOT, even in principle, be the ONLY method to acheiving the whole truth about the universe.

    I know that argument well too. It's BS - you need to show that there are things that are inaccessible to science rather than merely assert their existence. Nobody has ever done such a thing


    You might “know the argument well”, but it is apparent you do not understand it, NOR you seem to understand the (philosophical) basics upon which relies upon

    1 - First of all it is ON SICENCE the burden of proof that it can investigate EVERYTHING there is. It is not something one can take for granted.

    2 - Second one must show that even if it can investigate everything there is it can also explain it COMPETELY. This cannot also be taken for granted. Many problems (like the qualia problem) is an example of that.

    3 - Since science itself rests on SEVERAL premises that are NOT scientifically verifiable, already you have a problem

    4 – Most of science relies upon mathematics and in general mathematics is NOT something that is “empirically verifiable” through the scientific method but is a DEDUCTIVE science, rather than inductive like physics and biology.
    So if the cornerstone of science, mathematics, falls outside of science, you have another problem (sure one might say “”math is only a language, etc…”” but that does not cut it… there are people, even many scientists, who advocate mathematical Platonism i.e. that mathematical objects are real in some sense or another, not just ideas)
    Then again biologists are notoriously ignorant of mathematics (look at THIS epic fail: http://www.ncbi.nlm.nih.gov/pubmed/8137688, where this person goes back 300 years in the field of integrating functions!)

    5 – Science relies on mathematical MODELS often. It is NOT granted that such models truly reflect reality, but only parts of it… and that the mathematics behind a model can be deceiving (even some physicists, like Ellis and Smolin have raised this dilemma).

    ReplyDelete
  16. Ismael said...

    As I recall he does nto per se argue agains the idea of a minimal population... yet even if he did make a mistake there, so what?


    Apparently you don't understand the concept either. If you did, you would realized that the mistake he makes is of critical importance to his argument in that post. If he does not make that mistake and accurately represents the science instead, his post makes no sense.

    That's kind of important, don't you agree?

    Thay you really have NOT been reading properly.

    Intelligence Design is not the same a "teleology", certainly not in a scholastic sense.


    But he isn't defending ID, so how is that relevant?


    Sure evolutionary theory rejects A PRIORI some teleology, but this does NOT mean the teleology is not there.

    It's not a priori. Population genetics is pure mathematics - you have the principles of inheritance and within those rules, the rest basically follows directly as if you were doing pure math.

    Random sampling is key to most equations in the theory. And genetic drift plays a huge role.

    If the process has a direction of the kind that religiou philosophers have in mind, then the theory is false, because sampling is not random.

    It is, of cours, possible, that this is indeed the case, and things are teleological but indistinguishable from nonteleological. But that's a silly thing to propose - it's an ad hoc overcomplication of a model that does not need it at all.

    The theory opf evolution (which many SCIENTISTS thinks needs huge revisions: http://www.nature.com/news/does-evolutionary-theory-need-a-rethink-1.16080 a Nature article, note!) is simply a "model for reality".

    Sigh...

    This is why such pieces are doing more harm than good - flashy titles that mislead the people who either only read the titles, or are simply unable to understand what the issue is.

    Suffice ot say that nothing that needs "rethinking" (which is itself a highly questionable claim) changes the nonteleological nature of evolutionary theory.

    ReplyDelete
  17. Ismael said...
    1 - First of all it is ON SICENCE the burden of proof that it can investigate EVERYTHING there is. It is not something one can take for granted.


    This is laughable.

    There are a zillions imaginary entities of the same class as deities and souls that one can come up with.

    Someone needs to show you with 100% certainty that none of them exists?

    We study the things the existence of which we have good reasons to believe. So far all such things are accessible to science.

    If something appears that is both fairly certain to exist and inaccessible to science, then we can have a conversation. But no such thing is known.

    <3 - Since science itself rests on SEVERAL premises that are NOT scientifically verifiable, already you have a problem>

    This is laughable too. And religion rests on what?

    When am I going to see a believer apply the same epistemological standards that he applues to science to his own faith?

    The rest is just silly, some of it realy stupid ad hominem (you pick one example and generalize to all biologists - nice work, Mr. Sound Philosophical Reasoning), and I won't bother with it.

    ReplyDelete
  18. Oh, and something I forgot. Since the mathematical ability of biologists was questioned, it might be worth reminding that the conclusion that it is impossible for Adam & Eve to have existed came from the work of Li & Durbin on something called Pairwise Sequential Markovian Coalescent, which as the name makes clear, involves absolutely no math.

    And evolutionary biology is completely math-free too.

    ReplyDelete
  19. And the foundations of the modern theory of evolution was developed by Fisher, Wright and Haldane, who knew or used absolutely no math either...

    ReplyDelete
  20. Going back to the original issue of spontaneous decay (and to partially repeat what was said by another poster above), I've never really understood the issue here.

    Let's take an example: an electron in an excited state emits a photon and drops to the ground state. The various material causes don't really come into it; nor does the formal cause except in that it relates to the excited electron having the capacity to spontaneously decay with a certain half-life. The efficient cause of the photon and the ground state electron is the excited state electron; a final cause of the excited electron is the photon and ground state electron. Thus we have all four of Aristotle's causes.

    So either I've misunderstood the metaphysics, or the objection, or there simply isn't an issue here.

    The only way I can think of of violating Aristotelian causality is if one or more particles pop out of nothing. This doesn't happen, or if it does it isn't observable. (Before anyone starts citing spontaneous pair production from the vacuum in Quantum Field theory, I would say a) wholly disconnected diagrams don't contribute to perturbation theory, so such things can't in principle be observed even if they happen; b) one of the Feynman rules is the conservation of 4 momentum, so if we assume the positivity of mass and energy, the only particles which could be produced are zero momentum, zero mass and zero energy; c) the Quantum vacuum is not nothing.)

    ReplyDelete
  21. @BB

    Let's say that an electron transitions and emits a photon.

    Did this photon exist prior to being emitted by the electron?

    If so, where?

    ReplyDelete
  22. Bob said:

    Sure, and this is relevant to my comment how?

    He was asking how you handle the usual kind of counterexamples to your claim about simultaneous causation.

    ReplyDelete
  23. @Georgi Marinov

    Apparently you don't understand the concept either. If you did, you would realized that the mistake he makes is of critical importance to his argument in that post. If he does not make that mistake and accurately represents the science instead, his post makes no sense.

    You see I did not say he did not make a mistake. The problem is that that post was just a tentative idea not a theory he swears by or something.



    It's not a priori. Population genetics is pure mathematics - you have the principles of inheritance and within those rules, the rest basically follows directly as if you were doing pure math.

    Random sampling is key to most equations in the theory. And genetic drift plays a huge role.

    If the process has a direction of the kind that religiou philosophers have in mind, then the theory is false, because sampling is not random.

    It is, of cours, possible, that this is indeed the case, and things are teleological but indistinguishable from nonteleological. But that's a silly thing to propose - it's an ad hoc overcomplication of a model that does not need it at all.

    As you say “t is, of cours, possible, that this is indeed the case, and things are teleological but indistinguishable from nonteleological.”.
    No it is not a n overcomplication, since when we speak of teleology we do NOT seek to replace a scientific theory which a PRIORI denies teleology (you claim you do not rejected a priori… but it was historically rejected a priori, long before population genetics was even invented as a field).

    Sigh...

    This is why such pieces are doing more harm than good - flashy titles that mislead the people who either only read the titles, or are simply unable to understand what the issue is.

    Suffice ot say that nothing that needs "rethinking" (which is itself a highly questionable claim) changes the nonteleological nature of evolutionary theory


    Sounds to me like dogmatism. Clearly there are scientists who are NOT satisfied with the current affairs of things.
    And again: theories as just a set of equations can be non-teleological, but this does not mean that teleology cannot enter the playfield.

    ReplyDelete
  24. This is laughable.

    There are a zillions imaginary entities of the same class as deities and souls that one can come up with.

    Someone needs to show you with 100% certainty that none of them exists?

    We study the things the existence of which we have good reasons to believe. So far all such things are accessible to science.

    If something appears that is both fairly certain to exist and inaccessible to science, then we can have a conversation. But no such thing is known.


    The only laughable thing here is that YOU do not even read what I write before commenting. Did I say something about deities, souls or other things?
    The point is that there are many unexplained things in the world. Now many may and will be explained, but you cannot assume a priori that all that is unexplained CAN be explained (it might be, but you cannot assume it a priori).

    <3 - Since science itself rests on SEVERAL premises that are NOT scientifically verifiable, already you have a problem>

    This is laughable too. And religion rests on what?

    When am I going to see a believer apply the same epistemological standards that he applues to science to his own faith?

    The rest is just silly, some of it realy stupid ad hominem (you pick one example and generalize to all biologists - nice work, Mr. Sound Philosophical Reasoning), and I won't bother with it.


    Again you are being ridiculous. Except for some super-fundamentalists, people do not claim that a particular revelation explains everything you need to know. Even in late antiquity many philosopher and theologians praised “science” (or natural philosophy as it was the time).

    “When am I going to see a believer apply the same epistemological standards that he applues to science to his own faith? ”

    When am I going to see an ATHEIST apply those standards. It is clear you do not EVEN KNOW what the standards are, since you fail to sse some even common and basic critiques to scientism that even atheists (like Russell and others!) have raised.
    You do not even try to understand the objections. What I see from you is pure dogmatism in scientism… something that even many atheist philosophers have rejected.

    ReplyDelete
  25. , it might be worth reminding that the conclusion that it is impossible for Adam & Eve to have existed came from the work of Li & Durbin on something called Pairwise Sequential Markovian Coalescent, which as the name makes clear, involves absolutely no math.

    And evolutionary biology is completely math-free too.


    Actually those results have been debated among scientists (not talking here about religious debates), perhaps you chose to ignore such debate.

    What was that thing about standards you were saying…?

    ----------
    I think all I see here is a troll.

    Go back to your Dawkins shrine and pray for the salvation of your memes.

    ReplyDelete
  26. I need to go back and read Dowe. But as I recall, he also argues that we have direct evidence that spontaneous decay is caused: we can manipulate things in the lab, by photon bombardment, for instance, so that the chances of decay are much higher than they would otherwise be, which can only be interpreted as indicating that we are modifying the underlying causal situation. If spontaneous decay were genuinely uncaused, we shouldn't be able to do anything that would affect its probability of happening.

    ReplyDelete
  27. I'm going to hate myself in the morning for this, but for some reason I want to try.

    @Georgi
    It's hard to see how Feser makes colossal blunders in the post on evolution you cite when he grants every single modern finding of genetics and evolution. The entire point of that series was to argue (against a fellow "theist" as you would have it) that the findings of modern genetics and evolution don't contradict Aristotelian Metaphysics.

    ReplyDelete
  28. Bob,


    Did this photon exist prior to being emitted by the electron?


    Depends in part what you mean by `exist.' Aristotelian metaphysics has three states: actuality, complete non-existence, and in potentia. So some might define existence as either actual or in a state of potentia; others (like me) just count those things which are actual as existing.

    Those who accept the first definition might say that it exists virtually or potentially either `inside' (using the word metaphorically) the excited electron or the underlying quantum photon field. But using my preferred definition of existence, no it doesn't exist prior to the decay.

    ReplyDelete
  29. @Brandon

    He was asking how you handle the usual kind of counterexamples to your claim about simultaneous causation.

    There are no counter examples. Simultaneous causation is, as I said, incoherent.

    ReplyDelete
  30. There are no counter examples. Simultaneous causation is, as I said, incoherent.

    Historically people have proposed lots of counterexamples; he gave you one of them. In your comment to him, however, you concede this standardly given counterexample without any explanation how your claim about simultaneous causation is consistent with it.

    ReplyDelete
  31. @Brandon,

    He did not give me an example of simultaneous causation. So I am not sure what you are getting at.

    ReplyDelete
  32. He did not give me an example of simultaneous causation. So I am not sure what you are getting at.

    He gave you a standard example of simultaneous causation, a variant of one that was made especially famous by Kant.

    ReplyDelete
  33. @ Georgi Marinov

    Your entire corpus of writings seems to be one big exercise in pointing out how philosophy has conclusively shown that modern science cannot derive any knowledge with absolute certainty. Well, duh..

    The funniest part is that this is no part of Professor Feser's argument. He does not criticize science for reaching tentative, less-than-certain conclusions. (The Thomist William Wallace, even, has defended the position that science can reach some certain conclusions and that the thoroughgoing holism that most people attribute to science is overblown.)

    Rather his points are that: However successful science is, there are properly metaphysical conclusions, which are moreover necessary to ground the practices of science. And many scientific 'conclusions' about Thomism are rely on equivocations (i.e. on the meaning of 'cause') undergirded by unconscious metaphysics. Whether or not science can reach its conclusions with 'absolute certainty' is entirely beside the point.

    ReplyDelete
  34. @Brandon,

    Of course, as Kant probably would not have known, that is not an actual example of simultaneous causation...so?

    ReplyDelete
  35. Bob, what you think Kant "probably would not have known" is irrelevant, particularly given that you are clearly not sufficiently informed on the history of the subject you are pontificating about to recognize one of the most famous examples.

    Since you are so sure it is not an actual example of simultaneous causation, and are so confident of your ability to explain this that you can say that it is "of course" not one, despite being one of the standard examples of what people even mean by simultaneous causation, then you perhaps you can do Matt the courtesy of explaining exactly how it fails to be an example of simultaneous causation, and take a break from your usual intellectual laziness today.

    ReplyDelete
  36. @Brandon,

    It simply is not an example of simultaneous causation, regardless of how you feel about it, or me, for that matter.

    ReplyDelete
  37. Bob,

    In other words, you aren't actually able to give any kind of explanation to Matt because don't know anything about the subject and are pretending that you do. That's the only conclusion that can be reached on the evidence, given that we've already established that you're so ignorant of the subject as not even to recognize a variant of the most famous proposed counterexample to your claim in history, combined with your complete inability to explain why it is not a counterexample, as evidence by your refusal to give any explanation.

    ReplyDelete
  38. @Brandon,

    It is not a counter example because if it were, if in fact simultaneous causality was coherent, it would violate relativity, not to mention violating the definition of the words simultaneous and cause.

    ReplyDelete
  39. @Georgi Marinov
    It is, of cours, possible, that this is indeed the case, and things are teleological but indistinguishable from nonteleological. But that's a silly thing to propose - it's an ad hoc overcomplication of a model that does not need it at all.

    You will see the point of this "ad hoc overcomplication" if you make a tiny effort to look at the question from a point of view different from your own.

    If evolutionary theory was irreducibly teleological, thenit would in itself be an argument for the existence of God. We can all agree, I hope, that it isn't so.

    We can, then, ask the question: "Since evolutionary theory is not necessarily teleological, is it ALSO impossible for it to be compatible with teleology?" If it is, that would make it an argument against the existence of God.

    That's the point of the "silly proposition": it shows that, even if we grant that evolutionary theory is not necessarily teleological, it is compatible with a teleological reading. "Compatible" does not mean "necessary", of course.

    Why would someone then prefer the "ad hoc overcomplication" you criticize? Well, if someone knows, from independent arguments having nothing to do with evolutionary theory, that God exists, and he also agrees that evolutionary theory is true, he'll want to find a way to make the two truths compatible, as they must be. That's the usefulness of the "ad hoc overcomplication".

    ReplyDelete
  40. It is not a counter example because if it were, if in fact simultaneous causality was coherent, it would violate relativity, not to mention violating the definition of the words simultaneous and cause.

    Since this is so, I'm sure you can give us both (1) a derivation showing that a lead ball's causing a dent in the cushion to continue to exist, if simultaneous, would violate relativity; and (2) a precise explanation of how to understand the lead ball's causing a dent in the cushion to exist while denying the lead ball's causing is simultaneous with at least part of the dent's continuing to exist. (1), to be done, strictly requires a precise mathematical proof, since relativity theory is a mathematical system of equations; but you don't have to go into the full mathematics here -- just lay out the key steps in the derivation. (2), of course, is essentially what you need in order to answer Matt, and what you have repeatedly tried to avoid giving.

    Again, if you actually have an answer to Matt, give the answer, instead of taking the intellectually lazy route of handwaving on the very point of dispute. Matt says the head on the pillow is an example of simultaneous causation, because the head's being on the pillow is simultaneous with the pillow's being dented. You deny this. Now show it.

    ReplyDelete
  41. As far as the science goes, at least in the sense of 'design/intention' being discussed, the theory of evolution doesn't need 'undesigned' any more than it needs 'designed'. That goes entirely beyond the competence of science.

    And I'd like to know what portion of 'the entire corpus of' Ed's writings Georgi has even read.

    ReplyDelete
  42. Al said...

    We can, then, ask the question: "Since evolutionary theory is not necessarily teleological, is it ALSO impossible for it to be compatible with teleology?" If it is, that would make it an argument against the existence of God.

    That's the point of the "silly proposition": it shows that, even if we grant that evolutionary theory is not necessarily teleological, it is compatible with a teleological reading. "Compatible" does not mean "necessary", of course.

    Why would someone then prefer the "ad hoc overcomplication" you criticize? Well, if someone knows, from independent arguments having nothing to do with evolutionary theory, that God exists, and he also agrees that evolutionary theory is true, he'll want to find a way to make the two truths compatible, as they must be. That's the usefulness of the "ad hoc overcomplication".


    Good that you remdined me.

    So let's assume for a moment that all the philosophical and metaphysical arguments for the existence of a deity are valid. The best they demonstrate is the existence of a deistic god.

    However, you are not deists here, you are all Christians. And Christianity comes with an enormous edifice of very specific claims about the nature of the deity, that by no means follow from the philosophical and metaphysical arguments.

    Please elaborate on how the existence of the Christian god follows for those arguments. I have always wondered

    ReplyDelete
  43. @Georgi Marinov:

    "[L]et's assume for a moment that all the philosophical and metaphysical arguments for the existence of a deity are valid. The best they demonstrate is the existence of a deistic god."

    Actually they demonstrate the existence of the God of classical theism.

    "However, you are not deists here, you are all Christians."

    That's false as well. There are a number of posters here who are not Christian, including some who accept the arguments for classical theism.

    "Please elaborate on how the existence of the Christian god follows [from] those arguments."

    It doesn't. Who ever said it did?

    Now, if all you mean are divine attributes like omnipotence, omniscience, goodness (in a certain sense), simplicity, self-existence, and that sort of thing (about which there's nothing uniquely Christian), then they do indeed follow, and you'll find out what you want to know by reading the blog posts to which I just linked you. But if you mean the Trinity and Christ and so forth, I think you'll have a hard time finding anyone who ever seriously proposed that those could be established without recourse to revelation.

    ReplyDelete
  44. Hi everyone
    As a newcomer to Thomism, I recently had the chance to employ the first three ways in a debate with some atheist friends. The arguments were well received, but one of my friends attempted to circumvent them by saying such principles as cause and effect are known to us only because of what we experience in this universe, and we don't know whether they would apply to whatever reality there is outside of or before this universe. I responded that whatever exists, the crucial issue of the difference between existence and essence would still apply, and therefore the act/potency distinction would as well. Is there a better approach I should have taken?

    ReplyDelete
  45. Sometimes I feel like the blog should have a required reading list of particular posts. Georgi's objections with regard to teleology and scientism make it evident that an understanding of the basics is lacking. We are likely just going to end up talking past on another.

    ReplyDelete
  46. @Brandon,

    Strictly speaking, the lead ball is not the cause of the indentation. The cause is the gravitational pull of, for instance, the earth.

    This gravitational attraction is not a simultaneous cause, since the force of gravity Is limited the speed of light, therefore taking time, which of course does away with simultaneity with regards to the cause and again shows why simultaneous causality is, strictly speaking, incoherent.

    ReplyDelete
  47. BLS said...
    We are likely just going to end up talking past on another.


    That goes both ways

    ReplyDelete
  48. Strictly speaking, the lead ball is not the cause of the indentation. The cause is the gravitational pull of, for instance, the earth.

    This is manifestly false, since the same indentation will not exist without the lead ball even if the gravity is the same. If your conception of causation "strictly speaking" can't even handle basic causal counterfactuals, it isn't clear why you think this is a strict account of the cause.

    However, even if we ignore this, it doesn't establish what was to be proved. The gravitational pull of the earth is ongoing through time; it preceded the lead ball being on the cushion, it will continue after we take the lead ball off the cushion. Thus you have not eliminated simultaneity. Therefore you have not actually established what you claimed to establish.

    In addition, if we assume that gravity is traveling at the speed of light, the actual cause of the indentation even on your own assumptions would seem not to be the earth but the wavefront of the gravitational pull, as it reaches the ball+cushion system.

    the force of gravity Is limited the speed of light, therefore taking time, which of course does away with simultaneity

    This strongly suggests that you are confusing 'simultaneous' with 'instantaneous', which is not even correct English.

    ReplyDelete
  49. @Bob:

    "Strictly speaking, the lead ball is not the cause of the indentation. The cause is the gravitational pull of, for instance, the earth."

    The gravitational pull of the Earth doesn't cause all pillows to have spherical indentations, so even on your terms it seems that it's the gravitation pull specifically on the lead ball that should count as the "cause." Is the gravitational force on the lead ball not acting simultaneously (note that this doesn't mean "at the same dimensionless mathematical instant") with the ball's pressing into the pillow?

    ReplyDelete
  50. @Brandon:

    I think you got in ahead of me on every single point and then some.

    ReplyDelete
  51. @Scott

    No, Gravity does not act on the ball simultaneously. Gravity is limited by the speed of light.

    No gravity, no indentation.

    ReplyDelete
  52. @Brandon,

    There would be no indentation with out gravity. Gravity requires time. That is pretty much game, set, match...

    ReplyDelete
  53. @Brandon,

    Since simultaneous is defined as happening at the same time, my English is fine.

    ReplyDelete
  54. There would be no indentation with out gravity. Gravity requires time. That is pretty much game, set, match...

    If we were all idiots, I've no doubt that would be true. However, neither of your points actually addresses the argument -- you have a bad habit of ignoring any argument against you --

    first, because gavity's being a necessary condition for an event does not make it the cause of an event;

    second, gravity's "taking time" is not


    Since simultaneous is defined as happening at the same time, my English is fine.

    You previously said (and clearly implied again in your previous comment) that simultaneity was inconsistent with taking time. However, simultaneity is only inconsistent with complete succession, not taking time. Two ongoing events can be simultaneous with each other, for instance, even if they both take time. For instance, I can be cooking in the kitchen (takes time) while you are washing the dishes from the cooking (takes time) and these can be simultaneous, for instance, if I just keep handing dishes off to you as I continue cooking and you wash them while I cook and come up with more dishes for you to wash. This is because occurring at the same time does not require that they occur at the same instant. "Simultaneous" is not inconsistent with taking time, only "instantaneous" is. So, no, your English seems to be quite defective, since you repeatedly attribute to the simultaneous what belongs to the instantaneous.

    ReplyDelete
  55. "That goes both ways"

    With regards to teleology and scientism, Ed actually has written about the materialist metaphysics (brute facts, etc).

    I have no desire to debate here, but for reference, can you quote the problematic part(s) of the original sin post? I'm guessing it has something to do with just 2 of the individuals getting ensouled?

    It might also help to note that the Thomistic definition of teleology extends to anything that consistently produces a specific effect. Under this definition, even protons and electrons are teleological. It goes without saying that this definition isn't asserted, but rather it is argued for.

    ReplyDelete
  56. This comment has been removed by the author.

    ReplyDelete
  57. Just commenting to be emailed future comments. Carry on.

    ReplyDelete
  58. Sorry, accidentally deleted part of the comment when I went to post. It should read:

    second, gravity's "taking time" does not address the point I made about the gravitation wavefront, which you haven't addressed.

    ReplyDelete
  59. Bob,

    The necessary condition point could be pressed in a different way, by pointing out that while the lead ball making an indentation in the cushion requires gravity, it also requires the lead ball and the cushion. What is more, the conjunction or uniting of all these necessary conditions -- gravitational pull, lead ball, cushion -- is also, by the strictest logical necessity, a necessary condition of the indentation.

    Thus the criterion to which appeal in claiming that only gravitational pull should be considered the cause actually applies to multiple things in the scenario besides gravity; and to deny that any cause and effect can be simultaneous, you must on your own principle deny that the conjunction of lead ball, cushion, and gravitational pull from the earth is ever simultaneous with the indention caused by that conjunction.

    Given your penchant for trying to declare victory without actually responding to the arguments made, we'll see how long it takes for you actually to address this issue.

    ReplyDelete
  60. @Bob:

    "No, Gravity does not act on the ball simultaneously. Gravity is limited by the speed of light.

    No gravity, no indentation."

    Gravity is most certainly acting on the ball simultaneously with its pressing into the pillow; that's why it's pressing into the pillow.

    Perhaps you're overlooking Brandon's point about the "gravitational wavefront," which is a more sophisticated, thorough, and accurate version of my own. If the Earth suddenly winked into existence and there were a time delay before the gravity wave propagated to the ball and pillow, the ball wouldn't begin pressing into the pillow until the wavefront arrived. At any time at which the ball is pressing into the pillow, it's the force of gravity on the ball at that time that pulls it into the pillow. Hello, simultaneity.

    ReplyDelete
  61. To put it another way, you might as well say that a batter's being injured by a baseball pitch isn't "simultaneous" with his being hit by the baseball, because the pitcher's throwing the ball isn't "simultaneous" with the injury.

    ReplyDelete
  62. There is something very odd to saying one can describe probabilistically, with arithmetical certainty, completely random events.

    ReplyDelete
  63. Just to add my voice to the chorus once again...

    Doesn't gravity strike you as a strange example in your favor, Bob? Gravity cause the ball to fall into the pillow in the same way, as Scott illustrates, the pitcher is responsible for the injury to the batter in his throwing of the ball (an cause per accidens, after all). Gravity's acting on a body would be a choice example of a cause simultaneous with its effect.

    ReplyDelete
  64. English isn't my native language, so it might be some mistakes here and there.


    @Bob,
    If a cause can only be considered as such from the moment we have the effect, and an action can be considered the effect of a cause (an "immediate effect", I would say), then we can have simultaneous causation.
    We cannot have only "mediated" or "indirect" effects, for they require means by which they come into being, and these means would be effects of the indirect cause, and if they were indirect effects...... and so on, it is necessary then to exist "immediate effects", that is, the first effects that makes something to be a "cause".

    ReplyDelete
  65. wow, typos. My post should have read...

    "Gravity does not cause the ball to fall into the pillow in the same way, as Scott illustrates, the pitcher is responsible for the injury to the batter in his throwing of the ball (an cause per accidens, after all)."

    ReplyDelete
  66. Also the thing about indirect effects is only one possibility for simultaneous causation. I just used it to demonstrated that "simultaneous causation" isn't an oxymoron

    ReplyDelete
  67. @Georgi Marinov

    I've only briefly scanned through your comments. You seem a little confused about what kind of argument Ed is giving but at the same time are trying to give a front that you 'get it' that its a philosophical argument. Why keep conflating a metaphysical philosophical claim with a physical scientific one?

    In fact a lot of what you are saying is caricaturish, conflation and an oversimplification of the discussion.

    By the way, what is your Phd on if you don't mind me asking?

    ReplyDelete
  68. What exactly am I confused about?

    Enlighten me please.

    I have a PhD in biology

    ReplyDelete
  69. @ Georgi Marinov

    What exactly am I confused about?

    This statement:

    Your entire corpus of writings seems to be one big exercise in pointing out how philosophy has conclusively shown that modern science cannot derive any knowledge with absolute certainty.

    I am pretty sure Professor Feser does not care about that point.

    ReplyDelete
  70. Speaking of the basics, I think these might be relevant.

    http://edwardfeser.blogspot.com/2012/05/natural-theology-natural-science-and.html

    http://www.epsociety.org/library/articles.asp?pid=81

    http://edwardfeser.blogspot.com/2012/08/concretizing-abstract.html

    http://www.thepublicdiscourse.com/2010/03/1174/

    http://www.thepublicdiscourse.com/2010/03/1184/

    ReplyDelete
  71. Re: Georgi Marinov

    Georgi! What did Feser *just* say about evolution? This is neither the time nor the place!

    Oy vey.

    Evolutionary theory is non-teleological? Have you ever used the preposition "for" when describing some part of a living thing? Or the term "function" when speaking of physiology?

    ReplyDelete
  72. Curio said...

    Evolutionary theory is non-teleological? Have you ever used the preposition "for" when describing some part of a living thing? Or the term "function" when speaking of physiology?


    You forget the cultural context in which such expressions are used. It has only been a century or so since we have had sufficient reasons to abandon teleology, after millenia of complete dominance of the idea, and the message still has not reached the vast majority of people.

    "Function" in biology properly understood is nonteleological.

    ReplyDelete
  73. Greg said...
    @ Georgi Marinov

    What exactly am I confused about?

    This statement:

    Your entire corpus of writings seems to be one big exercise in pointing out how philosophy has conclusively shown that modern science cannot derive any knowledge with absolute certainty.

    I am pretty sure Professor Feser does not care about that point.


    Oh, I see. So you are making a distinction between "science can't prove everything with 100% certainty" and "there things that science cannot study".

    I am not.

    It's really the same statement.

    ReplyDelete
  74. Georgi, quick question: What do you think teleology actually is?

    ReplyDelete
  75. @ Georgi Marinov

    Oh, I see. So you are making a distinction between "science can't prove everything with 100% certainty" and "there things that science cannot study".

    I am not.

    It's really the same statement.


    You are confused in several respects.

    First, you said that Feser's claim is that "modern science cannot derive any knowledge with absolute certainty." You have now modified this to the very different claim that "science can't prove everything with 100% certainty."

    Next, those two statements are clearly not the same. The real obvious reason for this is you regard the first as obviously true ("Well, duh. Tell me something I did not already know."), whereas the second is literally offensive to you.

    But then the statements also obviously differ in meaning. One refers to the epistemic quality of those conclusions (perhaps tentative) that science does draw. The other refers to the possible objects of scientific study.

    ReplyDelete
  76. What a depressing waste of space most of this comment thread is. Two central casting trolls crapping up the combox with ill-informed, point-missing, question-begging brain droppings having nothing to do with the original post. I’m tempted to delete the whole thread, and if I hadn’t been at a meeting for most of the day I think I would have stopped it before it got started.

    Georgi alleges that I make “gigantic blunders” vis-à-vis evolution, at first offers no examples, then when pressed offers a non-example. He says that in a certain post I show that I don’t understand the “concept of effective population size,” but does not tell us exactly what it is I have gotten wrong. In other words, he resorts to sheer undefended assertion, exhibiting the very failing of which he falsely accuses others. Standard New Atheist shtick, of course.

    In fact, in the post in question I make no assertions about evolutionary biology at all other than citing some claims of Jerry Coyne which I concede for the sake of argument, claims I discussed in more detail in a follow-up post:

    http://edwardfeser.blogspot.com/2011/09/monkey-in-your-soul.html

    and where what I have to say merely summarizes points made by Kenneth Kemp and Mike Flynn. These posts are not even about the truth or falsity of the evolutionary claims per se, but rather about their alleged incompatibility with certain metaphysical and theological claims.

    But of course, Georgi is here just fishing for confirming evidence of the standard New Atheist “Philosophers and theologians don’t know ‘the science’” meme. Here’s how it works:

    Step 1, scroll through some book, article, or blog post written by a philosopher or theologian until you find a reference to some scientific topic;

    Step 2, read into that reference the most uncharitable interpretation you can come up with so that you can justify trotting out the standard “You don’t know the science” charge;

    Step 3, if you can’t find a way to interpret the reference in a way that would justify that charge, make the charge anyway;

    Step 4, ignore whatever it is the book, article, or blog post is actually about;

    Step 5, post the bogus charge in a combox somewhere;

    Step 6, high five the nearest fellow New Atheist and declare another victory for reason and science.

    Naturally, it only goes downhill from there. Georgi confidently assures us that the main theme of my work is that scientific claims are not absolutely certain. I don’t know what the hell he is talking about, and neither does he. In fact, of course, that is not even a theme of my work, let alone the main theme. His other remarks are equally clueless.

    (continued)

    ReplyDelete
  77. (continued)

    Then there’s Bob, who illustrates that other hallowed New Atheist tactic: Zero in on some passing reference to an idea you know absolutely nothing about, read into that idea some straw man you can attack, and cling tenaciously to the straw man no matter how many times it is explained to you that it is a straw man.

    On top of that, Bob doesn’t know what “simultaneous” means in the present context. It doesn’t mean “occurring at exactly the same thinly sliced instant.” I don’t know about Bob, but the rest of us are able to walk and chew gum at the same time -- that is to say, simultaneously. And these things are happening simultaneously even though they do not happen at an instant but over the course of many moments. No doubt Bob will tell us that I’m using “simultaneous” here in a commonsense way rather than in a technical, physics sense, which is exactly right. But precisely for that reason, there is not and cannot be a conflict between physics and common sense. There could be a conflict only if they were using the term in the same sense. (Of course, Susan Stebbing made this point five decades ago. Maybe in another five decades scientism-blinded dolts like Bob will finally get the point, though I wouldn’t hold my breath.)

    Bob also doesn’t know what “incoherent” means. It doesn’t mean “empirically falsified.” Even if simultaneous causation were empirically falsified (which it has not been, muddleheaded misinterpretations of the implications of relativity notwithstanding) it wouldn’t follow that it is “incoherent,” just that it is false.

    Anyway, as I’ve said, none of what Georgi or Bob have to say has anything to do with the topic of the original post. It’s that well-worn further New Atheist tactic: kick up as much irrelevant dust as you can when you have nothing to say in response to what your opponent actually said.

    ReplyDelete
  78. Georgi, Just a bit of advice.... you're well out of your depths here.
    You can't come to this combox and throw out those inane posts and not expect to be taken to task, well, you can.... if you're fine looking the fool. And, apparently you're fine with looking like a fool.

    Serious advice here: Kindly close your mouth for a moment and actually pay attention.

    ReplyDelete
  79. Georgi has a PhD in biology.
    I'm not stunned that biology at times is referred to as the soft science of butterfly collecting.

    ReplyDelete
  80. So you want details, here are some:

    This is what you wrote in that post I referenced:

    Supposing, then, that the smallest human-like population of animals evolution could have initially produced numbered around 10,000, we have a scenario that is fully compatible with Catholic doctrine if we suppose that only two of these creatures had human souls infused into them by God at their conception, and that He infused further human souls only into those creatures who were descended from this initial pair. And there is no evidence against this supposition.

    So basically you were told that the effective population size in the human lineage never went below 10,000 or so, and then proceeded to write 3400 words, at the end of which you completely ignored the scientific fact and went on with "there was an original pair from which we all came from", while maintaining that this is completely compatible with the science. It is not.

    This is why knowing the science matters (in this example, having some understanding of the coalescent), because a lot of the time it actually invalidates your arguments, no matter how elaborately you expressed them. And I gave you an example of you doing precisely that.

    ReplyDelete
  81. Dusty Beans said...
    Georgi has a PhD in biology.
    I'm not stunned that biology at times is referred to as the soft science of butterfly collecting.


    And theology is the hard science of what exactly?

    ReplyDelete
  82. Hi Dr. Feser,
    We all know that you're one of the best with allowing dissenting views from posting.
    No reasonable person would complain if you were to delete all of Georgi's posts. There's just no content there.
    Delete mine as well. This is an interesting topic and this combox, which rarely happens, has the elements more typical to AfterTheBarCloses crowd.

    Georgi Marinov is a non-contributor. Also, if you delete his posts I won't have to see that widow's peak on his head. Dear goodness, Georgi trim that peak back! You look like a fat version of that kid from the Munsters.

    Yeah, that's mean. Hopefully though you'll see this and then Feser will delete all of this nonsense.

    ReplyDelete
  83. Georgi, you brain dead mook.... whoever claimed theology was a science.

    You're not smart, pal.

    ReplyDelete
  84. The sycophants have come out of the woodwork...

    ReplyDelete
  85. Dusty Beans said...
    Georgi, you brain dead mook.... whoever claimed theology was a science.

    You're not smart, pal.


    Everyone for thousands of years

    ReplyDelete
  86. Whatever you want to tell yourself, fat boy.

    This is a good blog with some great discussion.
    I only lurk here. And I've seen some great posters who disagree with Feser. You're not one of those.
    You're a loud mouth, clueless clown.

    ReplyDelete
  87. Georgi Marinov's most previous post is a good example of uncharitable interpretation. From the quote alone, the numbers make absolutely no difference to Professor Feser's point vis-a-vis only two human-like creatures having souls.

    It is merely for the purpose illuminating the point.

    ReplyDelete
  88. "Everyone for a thousand of years"

    LOL.

    Prove it, chubby.

    ReplyDelete
  89. John West said...
    Georgi Marinov's most previous post is a good example of uncharitable interpretation. From the quote alone, the numbers make absolutely no difference to Professor Feser's point vis-a-vis only two human-like creatures having souls.

    It is merely for the purpose illuminating the point.


    It has no impact???

    There was no Adam & Eve. We know that

    That has no impact?

    And it absolutely does not matter that he basically reconciles that fact with his beliefs by completely ignoring it? And that he does that due to severe misunderstanding of the science?

    Anyway, I actually brought this up in the real context of the OP.

    If scientists are telling you that your philosophical claims make no sense because of such and such scientific fact, you are well-advised to listen to them rather than telling them how dumb they are for not having read Aquinas world-by-word and swallowed his metaphysics whole. The last thing you want to do in such situations is to show ignorance of the facts.

    ReplyDelete
  90. From http://edwardfeser.blogspot.com/2011/09/monkey-in-your-soul.html

    "For suppose we regard the pair in question as two members of this larger group who, though genetically related to the others, are distinct from them in having immaterial souls, which (from the point of view of Aristotelian-Thomistic philosophy and Catholic theology) are a necessary condition for the possession of genuine intellectual powers and can be only be imparted directly by God. Only this pair and their descendents, to whom God also imparts souls and thus intellects, would count as human in the metaphysical and theologically relevant sense, even if the other members of the original larger group are human in the purely biological sense. As Kemp writes:

    These first true human beings also have descendants, which continue, to some extent, to interbreed with the non-intellectual hominids among whom they live. If God endows each individual that has even a single [metaphysically] human ancestor with an intellect of its own, a reasonable rate of reproductive success and a reasonable selective advantage would easily replace a non-intellectual hominid population of 5,000 individuals with a philosophically (and, if the two concepts are extensionally equivalent, theologically) human population within three centuries. Throughout this process, all theologically human beings would be descended from a single original human couple (in the sense of having that human couple among their ancestors) without there ever having been a population bottleneck in the human species."

    I'd also like to see Georgi respond to Greg's most recent comment.

    ReplyDelete
  91. Georgi,
    I have no idea what your post has to do with anything.

    You're all over the place. Are you just joking around?
    Or maybe have autism or something??

    ReplyDelete
  92. Georgi,

    Seriously, did you even read the post you are criticizing? Or did you just scroll through impatiently until you found that one sentence, thinking you'd hit New Atheist polemical gold?

    Because if you had bothered to read the whole thing -- certainly if you knew anything about the larger exchange between Coyne, Flynn, and me that it was part of (including the follow-up post linked to above) -- then you'd know that the claim under consideration was NOT that the later creatures in question were descended from that "initial pair" alone. Rather, the idea is that they were descended from that (human-soul-infused) pair together with those other, biologically human (but non-soul-infused) creatures that made up the rest of the population of 10,000. That's the whole point of the thesis that Kemp, Flynn, and I were entertaining: that even if modern humans could not be traced to less than 10,000 individuals, this wouldn't be incompatible with the idea that there is, in a sense, an original couple IF we think of only some of this original population as having souls (initially two, then later more than two as souls were infused into those creatures who were descended both from those two and from those non-soul-infused but biologically human creatures with whom interbreeding took place).

    In short, you missed the entire point. But then, that was already obvious, and your not knowing what the hell you are talking about only becomes clearer every time you open your mouth. So, if I were you, I'd take Dusty's advice.

    But hey, if you wanna keep giving us rope, we're happy to keep hanging you with it.

    ReplyDelete
  93. I appreciated the topic of the post. I had thought quantum objections had more substance to them than they apparently do.

    ReplyDelete
  94. "If scientists are telling you that your philosophical claims make no sense because of such and such scientific fact."

    Is this serious?
    Science intentionally lacks that scope.
    A competing philosophy might be able to do that, but not the findings of science. Because those findings are interpreted within a philosophical framework.

    I can't believe I'm explaining this to Biology PhD. Or maybe I shouldn't be too stunned. He sounds like a new atheist type who typically rams heads with intelligent design fellas.

    ReplyDelete
  95. So basically you were told that the effective population size in the human lineage never went below 10,000 or so, and then proceeded to write 3400 words, at the end of which you completely ignored the scientific fact and went on with "there was an original pair from which we all came from", while maintaining that this is completely compatible with the science. It is not.

    Okay, some clarification here, please. Would it be accurate to summarize your argument essentially to the effect of:
    1. Ensouled humans are traceable back only through the population with which Adam & Eve and their decedents. (assumption)
    2. Every human alive today has a soul. (I know you don't accept this, but for the sake of argument)
    3. If ensouled humans are traceable back only to Adam and Eve, then we would also be able to trace back genetic characteristics (imprecise with the biology, I know) to them in the entire human population as well (since all humans have souls)
    4. We cannot do (3).
    5. Therefore, by reductio ad absurdum, premise 1, the assumption, is false.

    Perhaps a bit wordy and sloppy, but I think basically valid.
    Fair interpretation, or are you saying something entirely different?

    ccmnxc

    ReplyDelete
  96. Actually I did follow the discussion, and now you are backtracking

    Which is great, we're making progress.

    ReplyDelete
  97. Dusty,

    Let's keep it above the belt. Georgi's doing fine beating himself up. Just stand back and enjoy.

    ReplyDelete
  98. Anonymous,

    Well, ensouled humans is, I think, a misnomer. It follows from the definition of a "human" that they have a soul-part. That's why Dr. Feser is careful to qualify as "biologically human".

    ReplyDelete
  99. Actually I did follow the discussion, and now you are backtracking

    Georgi, you might wanna get some deodorant, 'cause I can smell the desperation from here.

    You need only glance up at the passage Anonymous @ 5:30 quotes above to see that what I'm saying now is what I said three years ago.

    I ain't the one doing the backtracking. Don't let the door hit you, and all that.

    ReplyDelete
  100. LOL!
    Georgi is a regular commenter over at Larry Moran's Sandwalk blog.
    Why am I not stunned to see that?

    Eddie Munster, ahem... Georgi, I have to say it again - you're out of your depths here.
    Don't feel bad, so am I.
    That's why I typically just lurk.

    But, I do know a fool when I see one (familiarity?), and you're that fool.

    You have mastered the paradoxical ability of saying a lot while saying very little.
    Go back to Sandwalk, regroup alittle, and come back with renewed.... nah, on second thought, stay there.
    Then you'll be unfettered with the groans of those not impressed with your lack of intelligence when you say things like "well the sigma factor of the polymerase.... NO GOD!"

    ReplyDelete
  101. Whoops.
    Just saw your comment now.

    I'll knock it off, Dr. Feser.

    ReplyDelete
  102. @ Anon

    I'd also like to see Georgi respond to Greg's most recent comment.

    I don't think he can, because the post I was responding to was indefensible, and he clearly didn't think about it for a full second before posting it.

    ReplyDelete
  103. Ed,

    Why has no-one organised a debate with you as a speaker yet? It’s gotta happen sometime.

    ReplyDelete
  104. As I said, we're making progress, and that's something to celebrate.

    It's worth noting where we started. If we were to have that discussion ca. 1250, the starting position would have been biblical literalism with respect to Adam & Eve being specially created, and the N_e being equal to 1 (because presumably, Eve would have either had the same genome as Adam because of that rib, or one Y chromosome would have been removed and replaced by an X).

    Many years much good science and a lot of goalpost moving later, we have a lot more constraints on the version of the story that fits the facts:

    1) Evolution happened
    2) There were two, either humans, or humanoids from an earlier species/chronospecies who were specially ensouled by the deity.
    3) The original ensouled pair interbred with non-ensouled humans, or presumably "subhumans"
    4) All the offspring of this lineage was ensouled
    5) The MRCA of all modern humans belonged to the ensouled ineage

    Why the deity chose to do this in such a complicated way, and why the story revealed in the holy book had to be so different from the truth is left unexplained, but then deities work in mysterious ways and who are we to ask so many questions

    Meanwhile my physics colleagues have been doing a lot of constraining on the properties of the soul, and by all reasonable criteria have basically constrained it out of existence. They have also shot down the premises of prime mover arguments and much of the rest of scholastic metaphysics, which has forced the supporters of the latter to try to, for example, devise new versions of quantum mechanics so that this is no longer the case.

    Given where we started, this is a lot of progress, hopefully there will be more in the future.

    ReplyDelete
  105. Dusty Beans said...
    "Everyone for a thousand of years"


    I claim to know my history of science.

    That you want to redefine science in such a restrictive way that it fits your goals is not my fault.

    LOL.
    Prove it, chubby.


    For the record, there has not been a single ad hominen attack in my posts here (saying that person X does not understand scientific concept Y is not an ad hominem attach), much less on completely irrelevant (and also factually wrong) to the subject grounds.

    ReplyDelete
  106. Georgi,

    "Why the deity chose to do this in such a complicated way, and why the story revealed in the holy book had to be so different from the truth is left unexplained, but then deities work in mysterious ways and who are we to ask so many questions"

    Did you really read all of the posts in the original sin conversation? Part 1, Part 2, Flynn's post, etc? This has been addressed.

    "Meanwhile my physics colleagues have been doing a lot of constraining on the properties of the soul, and by all reasonable criteria have basically constrained it out of existence. They have also shot down the premises of prime mover arguments and much of the rest of scholastic metaphysics, which has forced the supporters of the latter to try to, for example, devise new versions of quantum mechanics so that this is no longer the case."

    Why can't they post here? It would be more ontopic than the current discussion. And like I said, you have made it clear that you haven't gone over the basics of the arguments/metaphysics. Now we are into philosophy of mind territory:

    http://edwardfeser.blogspot.com/2011/05/mind-body-problem-roundup.html

    ReplyDelete
  107. It's worth noting where we started. If we were to have that discussion ca. 1250, the starting position would have been biblical literalism with respect to Adam & Eve being specially created,

    Even though we have guys like Augustine (who is no small figure in the Church) ca. ~380 saying no, we ought not (or at least are not obligated) to interpret Genesis literally. The goalposts had been moved long before modern science entered the picture.

    and the N_e being equal to 1 (because presumably, Eve would have either had the same genome as Adam because of that rib, or one Y chromosome would have been removed and replaced by an X).

    Didn't you just get done saying that we are no longer doing literal interpretation of Genesis (or have "progressed" from it)? If so, then why would be take it that God literally made Eve out of a rib of Adam (which many of us would take to be metaphorical or mythical)?

    Many years much good science and a lot of goalpost moving later, we have a lot more constraints on the version of the story that fits the facts:

    1) Evolution happened
    2) There were two, either humans, or humanoids from an earlier species/chronospecies who were specially ensouled by the deity.
    3) The original ensouled pair interbred with non-ensouled humans, or presumably "subhumans"
    4) All the offspring of this lineage was ensouled
    5) The MRCA of all modern humans belonged to the ensouled ineage


    Well, Feser and myself being Catholics, I completely reject your claims of goal-post moving, at least in any relevant sense. Perhaps you'd like to quote us some ecumenical councils or ex cathedra statements by a pope to the effect of us not being able to interpret books like Genesis in anything but a literal (or at least non-mythological) way. If you can, then goal posts have indeed moved. If not, then while what we say has changed, it is still totally compatible with the deposit of faith. We are merely adjusting with the views of science. Or are you now suggesting that's a bad thing (the whole heads I win tails you lose scenario)?

    ReplyDelete
  108. (continued)


    Why the deity chose to do this in such a complicated way, and why the story revealed in the holy book had to be so different from the truth is left unexplained,

    1. Well, considering God is outside of time and has no lack of resources, there is no particular reason that God would not choose such a way to bring about life. Further, delusions of omniscience aside, since we cannot know all outcomes (or possible outcomes) of any given mode of bringing about life, we are hardly in a position to be claiming one way is better than another. Sure, some might be simpler, but then, simpler in this context doesn't mean better.

    2. Are Holy Books somehow now required to be literal historical recounting of fact? Who cares if the story presented in Genesis isn't literally true. If it wasn't meant to be in the first place, I don't think there's much of a problem (not that you can't manufacture one).

    but then deities work in mysterious ways and who are we to ask so many questions

    And this is why I feel precisely zero pity when Dusty mocks you (by the way, what he says aren't ad homs; maybe I should be a condescending prick about your lack of knowledge of logic). But hey, who cares if you indulge in off-the mark chariactures as long as you get your fix, right?

    Meanwhile my physics colleagues have been doing a lot of constraining on the properties of the soul, and by all reasonable criteria have basically constrained it out of existence.

    I really wonder if you are just so used to your echo chambers that you don't realize you can't simply get away with making glib assertions like this. How does physics analyze something that isn't strictly physical, let alone disprove it? What tests have they run so as to discount the soul? Inquiring minds want to know.

    They have also shot down the premises of prime mover arguments and much of the rest of scholastic metaphysics, which has forced the supporters of the latter to try to, for example, devise new versions of quantum mechanics so that this is no longer the case.

    Well, you said at the beginning that you have little competence in physics, and you have now proved it beyond a doubt. Why you still thought yourself qualified to pontificate on the subject is anyone's guess. For such a supposedly rational person, you seem rather severely allergic to arguing for your points. But all that aside, where the hell have we been arguing for some new interpretation of quantum mechanics that saves causality. There has been some gesturing towards some non-local hidden variable theory, but that has been so minor that I can't even begin to comprehend where you got your claim from. But if you get to assert your conclusions so easily, I guess I can just as easily assert the existence of the Unmoved Mover.

    Given where we started, this is a lot of progress, hopefully there will be more in the future.

    Don't know who this "we" is, since you have consistently failed to grasp and address the general points we (as in, the Thomists) have been making. I'm not optimistic about the future either.

    ccmnxc

    ReplyDelete
  109. No, just sly insinuations about hypocrisy and dishonesty. This is a faintly grubby tactic called "willing to wound, but not to strike."

    "It's worth noting where we started. If we were to have that discussion ca.

    [...]

    5) The MRCA of all modern humans belonged to the ensouled ineage."

    The fact that past theories describing something were false, does not entail that current theories about something are false.

    I note that you have shifted to the above from "There was no Adam and Eve. We know that." You're backtracking, Mr. Marinov.

    "For the record, there has not been a single ad hominen attack in my posts here"

    Nor any arguments. We're waiting.

    [I hope Dusty forgives me for replying in his place].

    ReplyDelete
  110. Does the Scholastic tradition have anything to say about interpretations of probability?

    ReplyDelete
  111. @ Georgi

    Meanwhile my physics colleagues have been doing a lot of constraining on the properties of the soul, and by all reasonable criteria have basically constrained it out of existence. They have also shot down the premises of prime mover arguments and much of the rest of scholastic metaphysics, which has forced the supporters of the latter to try to, for example, devise new versions of quantum mechanics so that this is no longer the case.

    Oh, I see. Becoming an atheist, brb.

    ReplyDelete
  112. Georgi you are wrong if you think the Catholic Church conflates theology with science.

    Opposed to me arguing you wrong, how about you show how it did make such a conflation?

    I think much of this comes down to category errors you are making. You've made some wildly incorrect comments about how the data points of science should somehow guide the philosophy. That's coming at it from the wrong end. But, I don't think I want to give you that much credit. I really think you're just confusing what philosophy does, what theology does, and what science does.

    ReplyDelete
  113. "Meanwhile my physics colleagues have been doing a lot of constraining on the properties of the soul, and by all reasonable criteria have basically constrained it out of existence. They have also shot down the premises of prime mover arguments and much of the rest of scholastic metaphysics, which has forced the supporters of the latter to try to, for example, devise new versions of quantum mechanics so that this is no longer the case."

    I can't believe you have a PhD in biology.
    Or, you're just trolling the new atheists as a group.

    This can't be real.
    This has got to be another instance like the Blue Devil Knight who had bogus accounts where he would argue against a point he agreed with.

    Dguller, you're an atheist or agnostic. Put that quote from Georgi to the smell test. Is this a troll?

    ReplyDelete
  114. Or better yet: Georgi, you claimed to have read a decent amount of Ed's postings. You're commenting on this post so I'm going to assume you read it.

    So tell me; what was this post about?

    ReplyDelete
  115. This has got to be another instance like the Blue Devil Knight who had bogus accounts where he would argue against a point he agreed with.

    To correct something: what BDK did was pose as a Christian, but one who angrily condemned all anti-materialist/anti-atheist arguments as stupid and worthless, while telling Christians "people like you make me want to be an atheist!", and alluding to the GOOD arguments for Christianity and materialism which he typically A) refused to give, or when he did, B) were transparently lousy.

    Just to set that record straight.

    ReplyDelete
  116. As far as I understand, radioactive decay does not violate the Principle of causality. At best, the process is indeterminate and not random (more on that later).

    Here is why (according top my understanding anyway):
    Take the following representation of Silicon-32 changing to Phosphorus-32 via radioactive beta decay:
    http://hyperphysics.phy-astr.gsu.edu/hbase/nuclear/beta.html
    1) A neutron (n) in nucleus of Si-32 decomposes to a proton (p), electron (e-) and an antineutrino of the electron type (anti-nu(e)) via a W- boson. When Si-32 changes to Ph-32, a down quark in one of the neutrons of Silicon-32 spontaneously decomposes to an up quark (releasing an e- and anti-nu(e)), which in turn then changes the neutron (udd) to a proton (uud), which in turn changes Si-32 into Ph-32. The picture in the above link helps a lot.
    2) The process of beta decay (AFAIU) depends on the stability of the nucleus. A neutron in a nucleus will only decompose when the nucleus of an atom is unstable, meaning it has too much energy. This typically happens in atoms with a higher "neutron : proton" ratio. The reason for this is the interaction between the nuclear force and electrostatic forces. The nuclear force is especially attractive when nucleons (both protons and neutrons) are close to each other while the electrostatic force causes protons to be pushed apart. With too many neutrons, the nuclear force becomes too strong, causing the nucleus to become unstable with the position of the particles playing a causal role in the reaction (i.e. playing a role in mediating the weak interaction).

    3) The cause of the transmutation of n -> p is the weak interaction through Fermi coupling.

    4) The cause of the weak interaction can be described via weak isospin and weak fields. Weak isospoin is also a property of all particles according to the standard model. My understanding is as follows:
    a) The weak isospin of a quark generates a weak field.
    b) Weak fields interact with each other.
    c) This interaction generates a weak force.
    d) This weak force results in the weak interaction that mediates neutron decay.

    5) Statistically, the process occurs in a predictable manner. However it is in principle impossible to know the positions of all the particles (measurement of the particles will interfere with either position or momentum i.e. measurement problem) and thus not possible to know the exact moment each and every nucleon will decompose. So it is impossible to predict with 100% certainty when it will happen since we don’t know the location of all the nucleons before the event. Still, it can be statistically predicted with a certain amount of certainty. So, the process is indeterminate and spontaneous, but depends on these causal factors (weak isospin, weak interaction, instability of the nucleus etc.).

    ReplyDelete
  117. You may ask what is the difference between “indeterminate” and “random”? The way I see it is as follows:
    I acknowledge the fact that there is no agreed definition of randomness. However, one can perhaps make sense of the concept as an absence of ALL order or ALL predictability or the opposite of ANY order.

    Suppose there is something that behaved in a way that could only be described as random, something that changes in a totally unpredictable manner and acausal mannner. Let’s take an electron with spin Sz=+½ as an example. One moment it is an electron with Sz=+½ around the nucleus of hydrogen in laboratory on earth, the next moment it is moving towards the sun and randomly changes to a proton of carbon and then inexplicably moves back, the next moment it is some gold nugget on its way towards Mars. Suppose you want to measure Sz, you could never in principle know or predict whether it would suddenly change into a gold nugget or a proton or fly to the sun or Mars and back or just be Sx=+½ or Sx=-½ or not change at all etc. One can argue that such an electron behaves in a random manner as there is no way to predict any kind of behaviour. Randomness in this sense also implies acausality and I would argue that we have no observation of randomness anywhere.

    Contrast this with an electron that behaves in an indeterminate manner. Let’s take the electron with spin Sz=+½ again as an example. From experiments we know that Sx is indeterminate and that the electron is free to be either Sx=+½ or Sx=-½ upon measurement of Sx. We are able to predict that it will be either Sx=+½ or Sx=-½ even though it is indeterminate before measurement. The freedom is determined by something that is part of the electron, some property of the electron. One can say that the electron has certain dispositions, there is order (either Sx=+½ or Sx=-½, not pure radmoness) in the freedom of an electron. The freedom is not random, it is merely indeterminate. So while randomness entails indeterminism, indeterminism does not entail randmoness. One can have indeterminism and order (and thus causality)and one can have indeterminism and randomness but one cannot intelligibly argue to have pure randomness and order or orderly randomness.

    I would argue that atomic decay from this point of view is not random but merely indeterminate.
    Prime matter and pure potentiality also strikes me as indeterminate features of reality or “real being” (ens reale) so observations from modern physics should fit in quite nicely with Aristotelian metaphysics.

    ReplyDelete
  118. Georgi, there is not anything about evolutionary theory that states it is or is not teleological. Anybody that claims otherwise is injecting metaphysics into their empirical science.
    The nature of natural and evolutionary fitness is of course hotly debated. Sure, it is a philosophical question but it is important in determining whether evolutionary processes are teleological.
    Evolutionary fitness, as far as I understand, appear to play a central role in understanding the concept of natural selection. There appears to be at least two different views about evolutionary fitness:
    1) The propensity view of fitness.
    2) The statistical view of fitness.

    The propensity view, in a nutshell, views fitness is a probabilistic propensity/potentiality and it is a causal factor. This is very much compatible with Aristotelian metaphysics and teleology.

    The statistical view, again, in a nutshell, views fitness as a statistical probability and (as Alex Rosenberg argues) "deprives fitness of any causal or explanatory power". (http://www.science.uva.nl/~seop/entries/fitness/#FitSubPro)

    The debate between these views are interesting and ongoing. Here are a few articles if you are interested:
    Two ways of thinking about natural selection: https://tspace.library.utoronto.ca/bitstream/1807/17418/1/two_ways_of_thinking_about_fitness_and_natural_selection.pdf

    Selection and Causation (argues against a causal view)
    http://joelvelasco.net/teaching/167win10/matthen%20and%20ariew%2009%20-%20selection%20and%20causation.pdf

    Fitness and Propensity’s Annulment?
    http://members.logical.net/~marshall/AbramsAnnulment.pdf

    Fitness (Stanford Encyclopaedia)
    http://www.science.uva.nl/~seop/entries/fitness

    Matthen and Ariew’s Obituary for Fitness: Reports of its Death have been Greatly Exaggerated (argues for a causal propensity view)
    http://www.duke.edu/~alexrose/MAcomment.pdf

    What fitness can't be (argues against a causal view)
    http://faculty.missouri.edu/~ariewa/wp-content/uploads/2014/01/ARIEW-AND-ERNST-What-Fitness-Cant-Be.pdf

    Evolution is just change. If change and evolution are causal processes then it poses no problem for viewing them as teleological processes, even if the process is indeterminate. In fact, it fits in very nicely with Aristotelian notions of matter and form, potentiality and actuality etc.

    ReplyDelete
  119. Apologies for the many typos e.g.:
    The nature natural *selection of natural and evolutionary fitness is of course hotly debated.

    ReplyDelete
  120. Georgi, in case you are sincere, read and re-read the quote you posted. Read it again. And again. It never - not once - says what you are alleging. SERIOUSLY! In fact, it says the opposite! So the one example of Feser's supposed scientific illiteracy falls flat and just goes to show that you are being very hasty, and that's putting it charitably.

    You have a PhD, right? Then you should be beyond this childish New Atheist type BS. Conduct yourself better, man. Apologize for getting it wrong, and man up.

    ReplyDelete
  121. Georgi writes:

    As I said, we're making progress, and that's something to celebrate.

    It's worth noting where we started. If we were to have that discussion ca. 1250, the starting position would have been...

    Actually, Georgi, where we started this discussion was ca. 2:17 this morning, when you accused me of "gigantic blunders," "immense hypocrisy," etc.

    When pressed to provide evidence for these bold claims, you had nothing to offer but further hand-waving. Pressed again, you finally came up an "example" that blew up in your face and showed that you hadn't even bothered carefully to read what I'd written before shooting your mouth off.

    In other words, you committed a "gigantic blunder" and revealed yourself guilty of "immense hypocrisy."

    And that was just one of your assertions. Your other remarks have been similarly uninformed, similarly unsupported by anything but question-begging bluster.

    If you're really interested in "making progress," then instead of trying to change the subject -- as you are transparently doing now -- you might want to man up and just admit you were wrong.

    Then again, you've already told us that you don't like talking about stuff everybody already knows...

    ReplyDelete
  122. Here's my prediction: Someone will eventually cite my choice of the comic book illustration above as knock-down proof of my "scientific illiteracy" vis-a-vis the effects of radium...

    (Sorry if I beat you to it, Georgi!)

    ReplyDelete
  123. Oh dear. I've been busy and now I find an impenetrable forest of irrelevant comments regarding something something evolution something.

    Worse yet, my comment on "uncaused" radioactive decay has been pre-empted by Techne who had much useful description of the causal skein: http://edwardfeser.blogspot.com/2014/12/causality-and-radioactive-decay.html?showComment=1418451697507#c5488273713882253312

    I'll toss in Whitehead's comment in The Principle of Relativity that no event in physics is ever instantaneous, but always takes place over a period of time. Wooling over "instantaneous" thingies is a red herring.
    +++
    And since it appears my name has surfaced in its regard, I will only remind the statistically challenged that if there is a population of 10,000 ancestors at time T, then there is surely 2 ancestors at time T. Perhaps as many as 5000 such 2's. When we say that all the Connaughta are descended from Conn the Hundred-Fighter, we do not suppose that Conn was the only Gael west of the Shannon. When we say that all the Flynn cousins in this neck of the wood are descended from Martin of Loughrea, we do not deny that we all have sundry other great-great-great grandfathers.

    Consider so-called Mitochondrial Eve, who has a far more restrictive specification; viz., being an exclusively maternal ancestor. No one seems to think it is "double-plus unscientificalistic" that all human beings alive today are her descendants.

    ReplyDelete
  124. To note that the "minimum population" of 'original humans', strongly varies with the methods and parameters used for the calculation.

    for example in this articles it goes as low as 2000: http://www.genome.org/cgi/doi/10.1101/gr.6023607

    Other have criticized also these types of calculations based onb genetics alone:

    http://books.google.it/books?id=JZM6HplHIdMC&pg=PA9&source=gbs_toc_r&redir_esc=y#v=onepage&q&f=false

    http://www.ncbi.nlm.nih.gov/pmc/articles/PMC1449138/pdf/6799.pdf

    So one must be VERY careful when drawing conclusions from some pieces of research...

    ReplyDelete
  125. @Ed

    "Brain droppings" you know someone is going to abuse that right? Ha, love it.

    @Georgi

    It seems others have given a good enough answer. More in fact than I'm willing to devote time to.

    ReplyDelete
  126. @Dusty Beans

    I also disagree with Georgi on some points but please cut out the uncharitable ad hominem on him please.

    ReplyDelete
  127. @Dusty, Anonymous etc.

    Theology was indeed considered a 'science' in the historical sense of the word. Georgi is right on this point. Its material and focus is that of revelation.

    @Georgi

    I hope your Phd works out well. Biology is an important subject. I would charitably point out that Edward isn't bluffing. You do not realise your own assumptions and a priori ideas are currently getting in the way of understanding what he is trying to convey.

    trying to 'win' a debate online via aim-shoot-fire tactics isn't the best way to get to understand your opponents position - don't get me wrong, we can all be guilty of it sometimes. I would suggest having a listen. Peace.

    ReplyDelete
  128. @Ed

    Trump card. Here is the Comic.

    And Radium does this.

    So Na na nah, na na, I win!

    *notices Feser in before him* Oh. :(

    ReplyDelete
  129. John West,

    Does the Scholastic tradition have anything to say about interpretations of probability?

    I suppose it depends on what you mean. There is not, as far as I know, any general scholastic discussion of probability. But James Franklin has a good book, The Science of Conjecture, on the history of probability theory; a number of Scholastic moral theologians make a showing.

    ReplyDelete
  130. For what it's worth, on the Vox Nova site my main issue was with substance vs. accidents, which can't, it seems to me, be reconciled with atomic theory. Quantum mechanics has nothing to do with that. Every observable property of any material object except for its mass is sufficiently explained by the configuration of its valence electrons and the energy state of its molecules: color, texture, solidity, liquidity, gaseousness, conductivity, and so on. It's hard to see how there is any meaningful distinction between substantial and accidental forms, or how "form" in the Aristotelean sense is even meaningful, unless you either mean by "form" a "configuration of subatomic particles" or use the term of the properties of quarks.

    As to quantum physics and causality, I will have to read some more and give that careful thought.

    ReplyDelete
  131. Sorry for multiple posts, but the size constraints force it. Dr. Feser, you say the following:

    The quantitative description physics gives us is essentially a description of mathematical structure. But mathematical structure by itself is a mere abstraction. It cannot be all there is, because structure presupposes something concrete which has the structure.

    The eminent physicist Max Tegmark would disagree. His Mathematical universe hypothesis holds that external reality itself actually is a mathematical structure. I'm not sure if I agree with him or not, though the hypothesis is interesting; and I'd assume you'd disagree with him. The point is that it's not intrinsically absurd to say that mathematical structures are more than just abstractions. In fact, following Kurt Gödel, I'm a mathematical Platonist. That is, whether Tegmark is right or not about the universe, I believe that mathematical structures are not mere abstractions developed by humans, but real, objectively existing things that would be there whether humans existed or not, though they exist outside our world (or as our world, if Tegmark is correct).

    ReplyDelete
  132. For example, Einstein is commonly taken to have shown that our world is not really Euclidean. This could only be true if there is some concrete reality that instantiates a non-Euclidean abstract structure rather than a Euclidean abstract structure. So, physics itself implies that there must be more to the world than the abstract structure it captures in its purely mathematical description, but it does not and cannot tell us exactly what this concrete reality is like.

    I'm not quite clear on what you're saying here; but according to Einstein and Minkowski, the actual world we live in is the instantiation of a non-Euclidean structure. We just can't directly perceive what's beyond the Euclidean part of it. For example, if two-dimensional beings (à la Flatland) lived on the surface of a vast sphere, they would think they were on a plane, because any observations locally would differ so little from what you'd get in a flat surface that the difference would be undetectable. For example, lines of longitude would appear to be truly parallel, for a large enough sphere, though of course they'd eventually meet.

    Now if Flatland astronomers viewed very, very distant objects--say flat stars--they would notice that the measurements were inconsistent with a flat, Euclidean surface. For example, if two extremely distant objects were observed along "parallel" lines of longitude, it would be discovered that they were closer together than they should be (because lines of longitude converge). They could deduce from this that their Euclidan model was wrong and that they lived in a non-Euclidean world. That would be exactly true, since a positively curved surface such as a sphere is indeed non-Euclidean.

    Please note: 1. This would not be an abstraction, since the non-Euclidean curvature of their "Flat" land would be demonstrable empirically.

    2. The fact that this could be shown only mathematically and not actually visualized by 2-D beings doesn't mean their spherical world is not an instantiation of a non-Euclidean structure, since that's exactly what a spherical surface is.

    Finally, Dr. Feser, I admittedly spoke of you in somewhat strong terms on the Vox Nova thread. That was inappropriate, and I apologize. I do think the tone on some of your blog threads is rather pricklier than ought to be the case; but that does not excuse me. I do not claim expertise in either physics or Scholastic theology; but I do still think there are some major problems with Thomism-Aristotelianism, especially in regard to modern physics. I could, of course, be wrong. In any case, it is something for further study when I am less pressed for time.

    I end by saying that I'll look into the issues at greater depth when I have the time and resources to do so, and to say that while I'm still in strong disagreement with you on many of the issues here (pending the time to delve more deeply), I will try to make it cordial disagreement.

    ReplyDelete
  133. @Turmarion:

    "I'm not quite clear on what you're saying here…"

    I don't speak for Ed, of course, but his point as I understand it is just that if the physical world instantiates a non-Euclidean structure, then there must be something beyond that abstract structure that "has" it. And that just takes you back to your remarks about Tegmark and whether the physical universe can be just a mathematical structure.

    For whatever it's worth, I agree that mathematical structures are objective, discovered rather than invented, and independent of the human mind. But I think Thomism has that covered in holding, with Augustine, that such things "exist" (more precisely, "subsist") in the divine intellect. In this respect (and others) I regard Thomism as a successful synthesis of "neo-Platonism" and Aristotelianism, which are themselves just two different branches or streams of Platonism.

    ReplyDelete
  134. @Turmarion:

    "Every observable property of any material object except for its mass is sufficiently explained by the configuration of its valence electrons and the energy state of its molecules: color, texture, solidity, liquidity, gaseousness, conductivity, and so on."

    Suppose the wind along the shore happens to swirl the sand in such a way that it looks like a letter S. I happen to come along and see it, and when I do, I say, "Golly, that looks like the first letter of my name. Say, that gives me a swell idea! I think I'll take this handy stick and draw a letter S in the sand!" And so I do.

    In the latter instance the mark in the sand seems to me to be unambiguously a letter S, whereas in the first it doesn't. At the very least it seems coherent for someone to say that the first arrangement of sand isn't "really" a letter S (it just happens to look like one) and the second one is (because someone intended it as such). Does atomic theory explain the difference in the two cases, or do you not think there is one?

    ReplyDelete
  135. one must feel sorry for Feser here, I get the sense that something like 60% of the comments on this particular post are in bad faith. Better luck next time Professor!

    @grodrigues at 3:53am you warned them but they didn't listen, why didn't they listen!

    @Techne it seems like you have chosen the least helpful possible notion for the word "random", to the point that nothing random can actually exist. Anyway the issue isn't really about indeterminacy or probability, it is about cause.

    The way I would phrase the pivotal question is as follows. At some moment of time I notice that my radioactive lead has spit out of helium nucleus and turned into mercury. The interesting question is not "what caused it to turn into mercury?", because we know that this is what radioactive lead does sometimes. The question is "why did it turn into mercury just then as opposed to earlier?"

    I can think of several ways to answer the question but I hate them all :

    (1) the worlds of "lead right now" and "lead just a moment ago" are exactly the same. There can be no possible answer to the "why now?" question. "Now" is not even a meaningful term in the lives of lead atoms. So the critics of causation are right.

    (2) there is a hidden feature of the world which acted just now but not earlier. Unfortunately, between the principle of relativity and Bell's theorem, this hidden feature might be very far away or even in the future and so we will have no luck finding it. So the critics of causation are wrong but the principle of causality is useless.

    (2') there is a hidden feature of the world that can be found with a particle accelerator that is unreasonably large.

    (3) the principle of causality does not apply to questions about intervals of time, because a length of time is not a substance. So the critics of causation are wrong, on a technicality.

    (4) I have oversimplified the connection between causation and logical inference. I should not be trying to use the principle of causality to ask a "why ... because" type of question. The critics of causation are actually criticising something else.

    What am I missing?

    ReplyDelete
  136. (Note that by "letter S" I don't just mean the shape of that letter; I mean, roughly, really being a mark intended to represent a letter of the alphabet. It's all but impossible that the two marks in the sand have exactly the same shape anyway.)

    ReplyDelete
  137. Scott,

    For whatever it's worth, I agree that mathematical structures are objective, discovered rather than invented, and independent of the human mind. But I think Thomism has that covered in holding, with Augustine, that such things "exist" (more precisely, "subsist") in the divine intellect.

    Could you explain in further detail?

    Is this roughly the view that Platonists hold, but throwing out the Chain and leaving the One? In this case, when you write that perfect mathematical particulars exist in God's mind, do you mean in the sense that - as I understand - the Forms cheetah, dog, and monkey exist in the Form "mammal".

    ReplyDelete
  138. @John West:

    I mean that mathematical structures, like all forms, subsist (not "exist," which I deliberately put in scare quotes) in the divine intellect as exemplars or patterns and that they fully exist only as instantiated "in" created things. If physical space has a non-Euclidean structure, then it really has that structure. But considered in the abstract, simply as a mathematical structure, it's something more or less analogous to an idea in the mind of God.

    My main point here, in reply to Turmarion, is just that Thomists, too, can say that "mathematical structures are more than just abstractions." So-called mathematical Platonism isn't the only alternative to regarding mathematical structures as creations of the human mind.

    ReplyDelete
  139. Thanks, Scott. Sorry if I ambushed your comment to Turmarion.

    I've been trying to sort out whether Scholastic or Platonic realism is preferable.

    ReplyDelete
  140. @John West:

    No ambush at all, and no need to apologize. I just don't want to get too carried away with a side discussion (especially one in which I'm likely to run out of interesting things to say fairly quickly!).

    "I've been trying to sort out whether Scholastic or Platonic realism is preferable."

    This may help, especially the first five paragraphs. (I particularly appreciate that the contrast between Plato and Aristotle is characterized—I think correctly—as "transcendental idealism" vs. "immanent idealism," the latter being more or less what you mean by "Scholastic realism.") The conflict may, in short, be overstated.

    ReplyDelete
  141. WIth all genuine respect to Prof. Feser (no sarcasm intended), I do take great exception that we commenters are merely spewing "squid ink," and we do not know what we are talking about.

    We have some very educated individuals on that blog trained on the graduate level in a variety of fields (mathematics, physics, biology, history, etc.) and to equate us with the level of ignorance however properly assigned to pop atheist Jerry Coyne is going past acceptable exaggeration to pure calumny.

    While I appreciate substantive dialogue, this kind of rhetoric to fellow Catholics is just over the top and does no one any good.

    "[The princes] are guided by their passions rather than by reason. Therefore, I will take refuge with the scholars...Philosophy makes them supermen and theology makes them divine...Here too war reigns the all-powerful master...The Peripatetics wrangle with the Platonists...Their dispute arises...to such a degree that the violence of arguments leads to insult and insult to blows...exchanging venomous words...at the expense of the good name of his adversary."

    (Erasmus of Rotterdam, "Peace Protests," pg. 141)

    "When [Christ] calls himself a vine and his disciples the branches, does he express anything but solidarity? How monstrous to see one branch of a vine fight with another branch! Would it not be equally so to see a Christian fight another Christian?" (pg. 150)

    ReplyDelete
  142. @ Anon

    We have some very educated individuals on that blog trained on the graduate level in a variety of fields (mathematics, physics, biology, history, etc.) and to equate us with the level of ignorance however properly assigned to pop atheist Jerry Coyne is going past acceptable exaggeration to pure calumny.

    Coyne has graduate training in biology too.

    In any case, Professor Feser's issue was clearly with people who come across a sentence they don't like, interpret it as uncharitably as possible, go on to proclaim that the scholastic enterprise is thereby refuted, when they are equivocating on the important terms in question and are not addressing arguments on that precise point that Professor Feser has made elsewhere.

    If you haven't done any of that, then you're fine; Professor Feser's post was not directed at everyone who reads Professor Crub-Uribe's blog. If you have done that, then it doesn't matter how much education you have. One can miss the point and equivocate whether or not he has a PhD.

    ReplyDelete
  143. Eek, *Professor Cruz-Uribe

    ReplyDelete
  144. Thanks, Greg. I can certainly understand why that kind of behavior would rankle Prof. Feser. I must admit that I have read Feser and found him very helpful and enjoyable at times. It just seems to me that, while he might disagree with the argumentation, to denounce commentators personally seems a bit much. Admittedly, some did go after Feser a bit strongly, but we are not enemies.

    I feel like people, in general talk, talk past one another, and passionate feelings get unnecessarily in the way of that. And that's a shame, especially when they are members of the same Church. But, hey, it's tradition for Catholics so why not. It might just be that I am the type of person who instinctually avoids as many conflicts as I can.

    Happy Advent and a Merry Christmas to you and Prof. Feser.

    ReplyDelete
  145. No problem.

    I've just skimmed over Professor Feser's post. I can't find him "denouncing commentators personally." For the most part he suggests that their critiques begged questions, equivocated, and missed the point, and he provided evidence; consequently, Feser claims that their denunciations of the entire scholastic enterprise are too strongly stated given their quality of argument.

    The two comments that might be said to be "personal" are the one about the squid ink and the one about Jerry Coyne.

    Unfortunately, the combox discussion that ensues largely consists of a couple of Cruz-Uribe’s readers competing with each other to see who can emit the most squid ink...

    As readers of the post on Oerter know, this essentially just repeats the completely point-missing objection from Oerter that was the subject of the post, while ignoring what I said in the post in reply to the objection! The combox discussion goes downhill from there, with so many points missed, questions begged and crucial distinctions blurred that you’d think you were reading Jerry Coyne’s blog.

    I grant that no one likes to be accused of committing fallacies, but these are hardly insults or ad hominems.

    Merry Christmas to you as well.

    ReplyDelete
  146. Another excellent discussion. I especially appreciated your suggestion that radio active decay as well as Newton's law of inertia may conceivably be explained by the natural spontaneous action of a substance's nature via its form. In which case its efficient cause is not an accompaning and touching external cause but rather its creating cause. This of course is a notion totally in accord with a Aristotelian/Thomistic notion of nature/form and causality. Which, as you point out, was a theme stressed by Fr. James A. Weisheiple in Nature and Motion in the Middle Ages, a book I often refer to.

    Linus2nd

    ReplyDelete
  147. @Scott W.:

    "It must be Advent."

    Yes, I think you'll find that 13 December usually falls during Advent. ;-)

    ReplyDelete
  148. Hello Turmarion,

    Thanks for your response and for your apology, which I am happy to accept. You are correct that my tone in some blog posts and comments can be prickly. But I think you’ll find that that is only ever the case when I am responding to people who have themselves been behaving badly (e.g. extremely nasty and ill-informed public figures like Coyne, Krauss, Dawkins, et al., or people who have been snarky in misrepresenting by own views). I know that this style is not to everyone’s taste, but as I have argued several times in various posts, I think that sometimes -- not always by any means, but sometimes -- a polemical tone is not only justifiable but even called for. Anyway, if someone is just disagreeing, strongly but civilly, I think you’ll find I am not at all prickly in response.

    Anyway, let me say something in response to your various points:

    1. Re: substances vs. accidents, there are several issues here that I think you’re running together but which need to be disentangled. First, a background point: Keep in mind that historically, Aristotelians have argued that there are substantial forms at different levels of material reality. Most fundamentally, we have a fourfold distinction between inorganic phenomena, rudimentary or merely vegetative living things, animals, and human beings. Then, within some of these four hierarchically ordered classes, we have further distinctions between sub-classes. For example, in the inorganic realm there is stone, lead, gold, water, etc. etc. As you know, all of these everyday objects would have been regarded traditionally as having substantial forms and thus as true substances.

    But there are also, on the traditional view, lots of everyday objects that are not true substances but have only accidental rather than substantial forms. Tables and chairs, shirts and pants, watches and cars would be examples, as would random piles of stones or dirt. All of these things are mere aggregates of substances rather than true substances in their own right. For example, a table would just be an aggregate of different bits of wood and nails (say) where the wood and the metal making up the nails would be the true substances and have substantial forms. Being a table, being a nail, etc. would just be accidental forms that these substances have taken on. But at some level of analysis any such object will be composed of things that have true substantial forms, because accidental forms and mere aggregates presuppose true substances and substantial forms.

    Now, suppose that the traditional view about everyday medium-sized objects -- stones, lead, gold, etc. -- is wrong. In particular, suppose it’s true that, as you put it, “every observable property of any material object except for its mass is sufficiently explained by the configuration of its valence electrons and the energy state of its molecules.” It does not follow that there are no substantial forms. The most that this would show is that the true substances -- entities with substantial forms -- are to be found exclusively at the micro level rather than (as Aristotelians historically have supposed) also existing at the level of ordinary everyday objects. Being a stone, for example, would be an accidental form of the particles that make up the stone, with (some level of) particles being the entities that really have substantial forms.

    (continued)

    ReplyDelete
  149. (continued)

    The Aristotelian argues that there has to be some such level given the theory of act and potency (for reasons I explain in detail in my book Scholastic Metaphysics, for anyone who’s interested). Whether stones, lead, gold, etc. have substantial forms or are really just aggregates of their micro-level parts is a secondary issue. So to say “These everyday objects turn out not to have substantial forms but are reducible to their micro-level parts; therefore there are no substantial forms at all” is to commit what I characterized in the main post above as the fallacy of confusing a particular application of a principle with the principle itself. Again, Aristotelians never claimed that all ordinary objects had substantial forms in the first place. So the most that the considerations you appeal to would show is that the range of medium-sized objects that lack substantial forms but have only accidental forms is much wider than earlier generations of Aristotelians thought. It would show that Aristotelians were wrong historically about which things had substantial forms, not whether there are substantial forms.

    So that’s one point. But second, having said that, I think it’s not in fact correct to say that “EVERY observable property of ANY material object except for its mass is sufficiently explained by the configuration of its valence electrons and the energy state of its molecules.” For example, take consciousness, which is common to human beings and non-human animals, and which is defined by what contemporary philosophers call “qualia,” those aspects of experience that are knowable only from the first-person point of view. They are notoriously difficult to account for in reductive materialist terms. Indeed, I would say that it is impossible in principle to do so, given that modern science works with a purely quantitative notion of matter and qualia are (as the name implies) irreducibly qualitative. Yet non-human animals, at least, are purely material. That means that there are, in the case of animals, material features of the world that are irreducible to micro-level features -- just as the traditional Aristotelian view that animals are an irreducible kind of material substance maintains. What contemporary philosophers call “the qualia problem” is thus really just a partial rediscovery of what Aristotelians have always known.

    That’s just one example. One could argue for the irreducibility of lots of other material substances. Some contemporary philosophers of chemistry with no Scholastic ax to grind have argued that not even chemistry is really reducible to physics, even though it is commonly assumed to be.

    A third point is that the Scholastic metaphysician would hold that the micro-level parts are in a substance only “virtually” rather than “actually” in the first place, and in a sense depend on the whole as much as the whole depends on the micro-level parts. Spelling all this out and responding to the usual objections is not something I can do in an already lengthy combox discussion, though -- again, interested readers are referred to my book Scholastic Metaphysics.

    (continued)

    ReplyDelete
  150. (continued)

    2. I’m familiar with Tegmark’s views -- his recent book is on my to-read stack -- and as you surmised, I disagree with them. And as you’d also no doubt expect, I’m not exactly a Platonist about mathematics, at least if by “Platonism” we mean commitment to a “third realm” of abstract objects alongside the material and mental worlds. (If instead we mean an Augustinianized form of mathematical Platonism that takes mathematical objects to exist in the divine mind, I don’t reject that -- I think that’s basically correct.)

    But anyway, my remark was in fact neutral between Platonic realism and Aristotelian realism. Platonists and Aristotelians agree that we have to distinguish between abstractions and the concrete objects that instantiate the abstractions -- they just disagree about the ontological status of the abstractions themselves. My point was that the physical world cannot be merely an abstract structure but instead instantiates an abstract structure (whether we think of the structure in Platonic or Aristotelian terms). If Tegmark thinks that mathematical structure itself is all there is to the physical world, then I’ve got an abstract Platonic form of a bridge to sell him, because I think that just isn’t a coherent position.

    Anyway, physics itself, of course, can’t settle any of this, since we’re talking metaphysics now. And that was the main point in the passage from which you’re quoting -- that physics itself implies that there are truths about physical reality about which physics cannot even in principle tell us.

    3. Re: Einstein and Minkowski, I’m not sure how what you say conflicts with anything I said. Indeed, when you say “according to Einstein and Minkowski, the actual world we live in is the instantiation of a non-Euclidean structure,” that was precisely my point. There are several possible abstract structures -- Euclidean and the different non-Euclidean structures. The world instantiates one of these abstract structures but not the others. Therefore it cannot be right to say that the world just is an abstract structure. That would be a category mistake. Rather, the world is what has or instantiates the structure. But since physics only tells you about the structure itself, it doesn’t tell you everything about the physical world. In particular, it doesn’t tell you about the intrinsic, non-structural aspects of that which has the structure.

    ReplyDelete
  151. his recent book is on my to-read stack

    Weird question (and perhaps personal, so feel free to refrain from answering), but since you are quite obviously an absurdly busy person, and you seem to have wide reading interests (philosophy, theology, politics/culture, history, comics, conspiracy theories, etc.), how quickly are you actually able to get through all these books?

    ReplyDelete
  152. Hello Anonymous,

    Thanks for your comments. I wasn't talking about all the commenters at Vox Nova -- I said that I was referring to "a couple" of them.

    When someone badly misrepresents what I've said and then snarkily and dismissively attacks the straw man they've constructed, I do tend to respond very forcefully, and I think justifiably so. I'm used to this kind of thing from New Atheist types, and precisely because it was in this case coming from Catholics, I found it especially annoying.

    But anyway, I'm happy that that now seems to be water under the bridge.

    ReplyDelete
  153. @Ed,

    (If instead we mean an Augustinianized form of mathematical Platonism that takes mathematical objects to exist in the divine mind, I don’t reject that -- I think that’s basically correct.)

    In which case we'll take this as an opportunity to get in a call for that entry on the Proof from Eternal Truths you threatened years ago. Hint hint

    ReplyDelete
  154. Brandon,

    I suppose it depends on what you mean. There is not, as far as I know, any general scholastic discussion of probability. But James Franklin has a good book, The Science of Conjecture, on the history of probability theory; a number of Scholastic moral theologians make a showing.

    Thank you, Brandon. I'll add it to the to-read list.

    ReplyDelete
  155. @Dr. Feser,
    Newton’s principle of inertia, on this view, is a description of the way a physical object will tend to behave vis-à-vis local motion given its nature or substantial form.

    I don't want to rehash your argument with Oerter about angelic inertia, but the reason you proposed it as an option to begin with was because a finite cause isn't supposed to have a potentially infinite effect under some rubric of proportionality.

    This leads to my second point which is that if celestial circular motion can be "salvaged" as a local linear motion, and natural motion can be "salvaged" as a type of probabilistic causation in radioactive decay, why would I assume the rest of Aristotle's physics isn't similarly vague and flexible? If there is such a strong disconnect between his metaphysics and physics why is there any compulsion to salvage his physics if it has no bearing upon his metaphysics?

    ReplyDelete
  156. "The decay thus has a cause in the sense that (i) it has a formal cause in the nature or substantial form of the particular Pb210 atom, and (ii) it has an efficient cause in whatever it was that originally generated that Pb210 atom (whenever that was)."

    I have a problem with this. Didn't Aquinas say that whatever is moved is moved by another? So if the particle at t1 has not yet begun to decay, then it only has the potential to do so. Then, if at time t2 it begins to decay then this potentiality has become actual. But this requires something else to bring about this change, and the efficient cause that originally brought the particle into being is not sufficient for this. So how does this not go against Aquinas' principle?

    ReplyDelete
  157. Steven Jake,

    the efficient cause that originally brought the particle into being is not sufficient for this.

    What is your reason for saying this? It's what the (ii) in the section you are quoting is denying. Aquinas himself did in fact interpret the principle in such a way that that generating causes could be recognized as the movers for some kinds of effects arising from the natures of the generated things.

    ReplyDelete
    Replies
    1. Brandon,
      But an efficient cause is simultaneous with its effect. And whatever brought the particle into being does not seem to be simultaneous with the particle decaying. So how then can the efficient cause that generated the particle also be the efficient cause of the decay, especially since this is a casual series ordered per accidens?

      Delete
  158. Hi Steven,

    If you look at the Summa Contra Gentiles, you will see Aquinas' definition of the principle in paragraph 3.

    http://dhspriory.org/thomas/ContraGentiles1.htm#13

    Paragraph 9 seems to support what you are saying:

    "[9] In the third way, Aristotle proves the proposition as follows [VIII, 5]. The same thing cannot be at once in act and in potency with respect to the same thing. But everything that is moved is, as such, in potency. For motion is the act of something that is in potency inasmuch as it is in potency. That which moves, however, is as such in act, for nothing acts except according as it is in act. Therefore, with respect to the same motion, nothing is both mover and moved. Thus, nothing moves itself. "

    However, if you look back at paragraph 8, you will find that Aquinas felt the generating efficient cause that produced the substance was a sufficient mover for any causality stemming from that substances causal powers to justify upholding the principle. Although paragraph 8 takes some parsing.

    So even if T2 is truly spontaneous and without a cause, it still has a cause in the efficient cause that produced the substance.

    Hopefully I haven't butchered Aquinas too much here.

    Cheers,
    Daniel

    ReplyDelete
  159. @John West and anyone else interested:

    On the subject of forms as ideas in the divine intellect and Aquinas's synthesis of Plato and Aristotle on the matter, here's a book that came out about three months ago. I just now happened across it so I haven't read it yet, but from the samples it looks quite good. I'll be buying a copy at some point.

    Has anyone read it already?

    ReplyDelete
  160. @Steven Jake:

    "But an efficient cause is simultaneous with its effect."

    The efficient cause of the Pb²¹⁰ was simultaneous with (the creation of) its effect, namely the Pb²¹⁰ itself. So long as the decay is in some way brought about (deterministically or otherwise) by, or as a consequence of, the nature of Pb²¹⁰, the efficient cause of the Pb²¹⁰ doesn't have to be simultaneous with the event of the decay in order to count as its efficient cause.

    ReplyDelete
  161. Thank you for your response, Dr. Feser.

    I don't think all phenomena are reducible to subatomic interactions. The human mind, for example, is not purely a product of material forces. As a theist and in fact a Catholic, I don't doubt that. Life, broadly speaking, may or may not be fully reducible to physical phenomena. I incline towards saying that it's not, but I'm rather agnostic about that question, since I think we don't have enough understanding of it right now. I would assert that for nonliving things, and for those properties of living things not directly pertaining to their life, all properties are reducible to subatomic phenomena, and that thus there is no substantial form of "rockness", for example.

    The most that this would show is that the true substances -- entities with substantial forms -- are to be found exclusively at the micro level rather than (as Aristotelians historically have supposed) also existing at the level of ordinary everyday objects. Being a stone, for example, would be an accidental form of the particles that make up the stone, with (some level of) particles being the entities that really have substantial forms.

    Well, I can agree with that. As I said in my previous post, I could see the "flavor", "color", and fractional electrical charge of a quark, or the charge and mass of an electron, as being their substantial forms, and then "rockness", for example, being an accidental form. Where I'm coming from is that I've seen some AT philosophers who seemed unwilling to consider this.

    ReplyDelete
  162. (continued) As to the 4-D universe, I guess we agree that the cosmos is the instantion of a 4-D reality. Your phraseology threw me. You said, "For example, Einstein is commonly taken to have shown that our world is not really Euclidean." Your usage of "commonlly taken" implied to me that you thought he was wrong. Then you said, "This could only be true if there is some concrete reality that instantiates a non-Euclidean abstract structure rather than a Euclidean abstract structure." That confused me, because it seemed obvious to me that "Einstein showed that our world is not really Euclidean" means the same thing as "Our world is a concrete reality that instatiates a non-Euclidea abstract structure."

    A third point is that the Scholastic metaphysician would hold that the micro-level parts are in a substance only “virtually” rather than “actually” in the first place, and in a sense depend on the whole as much as the whole depends on the micro-level parts.

    In one of his books, the late, great Mortimer Adler--for whom I have great respect, though I disagree with him in many areas--said this, i.e., that atoms exist only in potentia. I have to disagree strongly with this. That would be like saying the letters of a word are only potential, existing only if you cut the paper so they're all separate!

    Regardin quantum mechanics, as I said, that's not my expertise. I think it would be fair to say that some quantum events seem difficult to account for given traditional metaphyics. I understand what you say about burdens of proof, and it's true that all kinds of metaphysics have been read into QM. However, anomalous phenomena, it seems to me, do have to be accounted for. When Becquerel found that radium fogged photographic film in the dark, he didn't think, "Well, the general principle that light exposes film is still obviously sound, so I don't need to worry about this." Rather, he and others studied this and discovered radioactivity. I don't think anyone denies that causality and such work in most everyday situations. I do think it is perhaps a bit much to say that anyone who disagrees with traditional metaphysics has to come up with an entirely new metaphysical system before he critiques the old one. It would be as if someone said, "Only light can expose film," and the Becquerel said, "But what about radium fogging film in the dark?" He did have do the work to demonstrate radium; but even before that, would it have been unreasonable to think that maybe the traditional framework was at least incomplete? I don't think critics are necessarily trying to trash Thomism-Aristotelianism completely; they're just saying that it doesn't seem to have the tools to explain some features of modern physics. Maybe no one has such tools yet.

    As to mathematics (and philosophy more generally) I am indeed a Platonist. Whether mathematical truths are in a third realm or ideas in the mind of God, I don't have a strong opinion. Even in the first case, they'd derive ultimately from God. I'm not sure that we're not ideas in the mind of God actually; but that's another issue.

    ReplyDelete
  163. Scott: What I meant when I said that every observable property of an object is explained by its molecular and atomic configuration, what I meant was that there are no substantial forms at the macro level. There is no such thing as "rockness" or "woodness". Rather, all properties of a rock or a piece of wood are emergent phenomena ultimately derived from the masses and charges of sub-atomic particles. As you see in his response, Dr. Feser seems to acknowledge that this is a possible interpretation, though I'm not necessarily sure whether he subscribes to it:

    [S]uppose it’s true that, as you put it, “every observable property of any material object except for its mass is sufficiently explained by the configuration of its valence electrons and the energy state of its molecules.” It does not follow that there are no substantial forms. The most that this would show is that the true substances -- entities with substantial forms -- are to be found exclusively at the micro level rather than (as Aristotelians historically have supposed) also existing at the level of ordinary everyday objects. Being a stone, for example, would be an accidental form of the particles that make up the stone, with (some level of) particles being the entities that really have substantial forms. (my emphasis).

    That was my point. I'd agree with you that mind--which cannot be accounted for by only physical and chemical phenomena--is necessary to explain how the "s" you draw is different from the "s-like" line produced by the wind and waves. My point wasn't to deny that; only to say that any substantial forms must be at the micro level, which is indeed contra the traditional Aristotelian supposition.

    I mean that mathematical structures, like all forms, subsist (not "exist," which I deliberately put in scare quotes) in the divine intellect as exemplars or patterns and that they fully exist only as instantiated "in" created things.

    For reasons I won't go in to here, but which I've discussed a bit at this post on my blog, I'm not quite sure that there is much distinction between "existing" and "subsisting in the mind of God" when you get right down to it. I used to think Bishop Berkely was cracked in his metaphysics, but now I'm not so sure. Maybe esse est percipi--where God is the one who perceives--really is right.

    ReplyDelete
  164. Steven,

    As Scott notes, the relevant simultaneity is in the actual generation. And generation of natures is a per se, not per accidens, ordering.

    The basic idea of the suggestion is that spontaneous decay works like spontaneous movement to natural place. (It has been recognized since Pierre Duhem noted it a hundred years ago that equilibria in modern physics structurally and functionally work in explanation like natural place did in Aristotle.) And thus the idea is that the radioactive active substance is generated in such a way that it has a natural tendency to a certain stability point, an equilibrium or 'natural place', and thus, as a result of its nature, tends spontaneously toward that stability point, all other things being equal. Since this is exactly how Aquinas himself thought of the natural motion of elements, and, as Daniel notes, he explicitly discusses the natural motion of elements in discussing the general principle, it's clear that an explanation of this general type is consistent with how Aquinas himself understood the general principle.

    We have to be careful in talking about simultaneity when discussion Aristotelian accounts of causation. First, the simultaneity in question is simply a kind of unity or co-actuality (most basic kinds of causation the cause's causing and the effect's being caused are one and the same action) and thus is not primarily understood in terms of time at all, although in fact we do tend in most cases to recognize simultaneity by temporal measurements. (The difference becomes relevant, though, for considering the sense in which God's causal action is simultaneous with His effects.) Second, while Aristotelians think that the most basic forms of causation are simultaneous with the effect, they don't claim that all genuine causation is simultaneous with the effect -- it's just that you can further analyze other kinds of causation into simultaneous causation plus various other things. Third, Aristotelians on their own don't tend to emphasize simultaneity all that much; it comes up occasionally, but less often than one would think from these kinds of discussion. It comes up a lot these days because most of the alternative accounts of causation worth taking seriously are Humean, and thus deny that cause and effect can be simultaneous at all; because of that, it's a point Humeans will tend to attack and Aristotelians will tend to defend at length. But most of the importance arises not because of what it does for Aristotelians, but because it just happens to be a point on which the major positions on causation are directly opposed. Within the Aristotelian account itself it follows quite trivially from other things which are much more important.

    ReplyDelete
  165. @Turmarion:

    "I used to think Bishop Berkely was cracked in his metaphysics, but now I'm not so sure."

    Yeah, I don't think Berkeley was cracked at all. Honestly, I think his most significant problem is that in his earlier work he tended to write as though he was following Locke in identifying sense impressions with ideas; in his later work he's more explicitly Platonic.

    ReplyDelete
  166. Brandon writes: "As Scott notes, the relevant simultaneity is in the actual generation. And generation of natures is a per se, not per accidens, ordering."

    And just to amplify this in order to bring out what I take to be Ed's main point: the nature thus generated is itself what grounds the probability that the lead isotope will decay within a given unit of time. Once the efficient cause has generated this nature, the coin toss is already set in motion, so to speak; it's in the nature of the isotope to decay (perhaps nondeterministically, but not for that reason acausally).

    ReplyDelete
  167. @Turmarion:

    That right there is a mighty fine blog post. And I'm happy to see someone put in a good word for Rudy Rucker's Infinity and the Mind.

    ReplyDelete
  168. Hmm. When I previewed that last post, I got a page that had a "Publish Your Comment" button but no CAPTCHA, and I was able to submit my post without proving that I wasn't a robot.

    I've just done it again.

    ReplyDelete
  169. Regarding probability:
    Probability cannot be defined in the absence of a prior model. The probability that the next heat of steel will have a N2 content exceeding 90ppm cannot be answered without some assumptions regarding how nitrogen content is distributed from heat to heat. Probability simply cannot be elevated to an entity in itself.
    Regarding particles:
    When we say that electrons in valence shells account for everything, we forget that an electron cannot be in a valence shell unless something higher than the electron already exists; viz., an "atom." Such an electron behaves very differently from a free electron, so we cannot even predict electrons from electrons! Similarly, protons are + charged and so repel one another. Yet in a nucleus, many protons are often packed very closely together, so protons in a nucleus behave very differently from protons on the loose. That is, the parts are constrained by their membership in a natural whole.
    Regarding causation:
    For A to cause B need not mean that precise time and place of B can be predicted. "B is caused"≠"B is predictable." After all, Newton's laws do not predict which apple will fall from which tree and when.

    ReplyDelete
  170. Turmarion and Scott,

    Any Rudy Rucker fan is my kind of guy.

    Re: "commonly taken," Turmarion, I put it that way just to leave open the question whether to give relativity an instrumentalist reading, in case someone wanted to make an issue of it.

    Re: QM, I agree completely that one should not be glib in commenting on its unusual aspects. I also agree that it is not reasonable to expect someone never to say anything philosophical at all about it unless one has an entire worked-out metaphysical system. My point is that it is precisely those who would dismiss the principle of causality on QM grounds who are guilty of glibness. (Not that you and Prof. Cruz-Uribe were doing that -- I understand you were both just raising the question.) Dismissing the principle of causality is so far-reaching in its metaphysical implications that one ought, in that case, to have a general metaphysical picture before drawing such a conclusion. Especially if one accepts causality elsewhere, because it's very hard to make it coherent to reject it in one domain and not others.

    Scott, yesterday I saw a CAPTCHA that was nothing but a couple of black smears against a yellow background. Rather... indeterminate, like prime matter. Spooky, no? Fortunately I didn;t need to type it in in order to post a comment at my own blog.

    ReplyDelete
  171. @Turmarion:

    TheOFloinn writes, "Such an electron behaves very differently from a free electron, so we cannot even predict electrons from electrons!"

    Although this isn't really TheOFloinn's point, it's worth noting that this is an example of why Scholastic metaphysicians say, as Ed put it above, that "micro-level parts are in a substance only 'virtually' rather than 'actually'." It's not at all like saying that letters exist only when you snip words apart.

    To elaborate on TheOFloinn's example, hydrogen can't be "in" a chemical compound unless something higher-level than hydrogen already exists—water, for example. The properties and behavior of hydrogen "in" water are very, very different from its properties and behavior when it's flying solo. That's why some (including me) say that hydrogen is present in water "virtually"—if we do stuff, we can get hydrogen out of water—but not "actually."

    That doesn't mean that hydrogen just isn't "in" water at all; it's merely a recognition that hydrogen "in" water doesn't manifest the usual properties of, well, hydrogen. The disanalogy with letters and words should, I think, be clear.

    ReplyDelete
  172. @Edward Feser:

    "Any Rudy Rucker fan is my kind of guy."

    Heh. Now about half of us are going Yessss! and the other half are going Who? I'd better look this guy up. Was he in Steely Dan?

    "[Y]esterday I saw a CAPTCHA that was nothing but a couple of black smears against a yellow background. Rather... indeterminate, like prime matter. Spooky, no?"

    Now that you mention it, I recall seeing a couple of those too. I never expected to have a sensory experience of pure potency, but hey, I couldn't have picked a better or more appropriate place than your blog for it to happen.

    But yeah, something's gone wonky somewhere.

    ReplyDelete
  173. @Ed

    Surely they would have got the point here?


    A. Conflating empirical and metaphysical issues: Those who know some science but not a lot of philosophy very often assume that when a Scholastic philosopher says something about the nature of causality, or substance, or matter, or the like, then he is making a claim that stands or falls with what physics tells us, or at any rate should stand or fall with what physics tells us. But this is a category mistake. Scholastic metaphysics is not in competition with physics, but approaches the phenomena at a different (and indeed deeper) level of analysis. Its claims do not stand or fall with the findings of physics, any more than the claims of arithmetic stand or fall with the findings of physics. Indeed, like arithmetic, the basic theses of Scholastic metaphysics are (so the Scholastic argues) something any possible physics must presuppose.


    That being said I can see why someone might find QM problematic for causal relations (say a first year Physics student).

    Very much related to QM type objections to causality - I also see why the whole Newtonian concept of motion is used along the same lines.

    "Lex I: Corpus omne perseverare in statu suo quiescendi vel movendi uniformiter in directum, nisi quatenus a viribus impressis cogitur statum illum mutare."

    It goes somewhat like an object will stay in its state of motion or rest until acted upon (as you know anyway of course - you wrote on it somewhere if my memory serves me right). I've always thought on that front that there is the ignoring of the conservation of energy - so even from the science alone end there is a slight degree of picking and choosing so as to make an argument - a weak argument. Then we start taking in to account the convertibility of matter to energy etc. and they start running in to big problems. What I'm saying is the 'energy' never ceased to 'act' upon the matter and that the energy has something which caused it to act etc. (obviously in this case I am confining my point to a temporal/material type causal series).

    On another note have you ever compared schools of Thought in relation to Metaphysics within Thomism? It was just on my mind to ask you that since your kind of the 'go to' guy on Thomistic Metaphysics in English these days (which is a compliment I suppose).

    ReplyDelete
  174. Our concepts regarding causation remain very highly nuanced and rigorously defined and require significant disambiguation. Our descriptive sciences and normative philosophies remain methodologically autonomous and categorically distinct and shouldn't be facilely conflated.

    When we don't disambiguate our concepts and distinguish our categories, we can unwittingly engage scientism on its own confused turf. Dr. Feser's article was refreshingly spot on.

    In recent years, emergentist heuristics have fruitfully relied on Aristotelian-like notions of causation, including both formal and final causations (e.g. Baldwinian evolution, biosemiotics, morphodynamics, putative coevolution of language and brain).

     For example, in biosemiotics, downward causations include both those constraints derived from a reality's initial, boundary and limit conditions, which exert formal influences, i.e. morphodynamics, as well as those derived from mereological (part-whole) conditions and intentional realities (semiosis or interpretation), which exhibit final causations, i.e. teleodynamics.

     There's nothing inherently controversial about this essential emergentist paradigm as long as we recognize that the idea of emergence is mostly a heuristic device, which, like any good metaphysic, will have some descriptive accuracy but only limited predictive utility, hence, lacking explanatory adequacy. That is, it predicts novelty but cannot specify its nature. This is also to say that, properly considered, it wouldn't a priori specify whether this or that downward causation would necessarily (or not) violate physical causal closure, hence would be compatible with diverse philosophies of mind, would be consistent with competing ontological perspectives, e.g. reductionisms and dualisms. For example, while many emergentists invoke notions like supervenience, it's clearly problematical, trivial when described as weak (and usually associated with strong emergence), question begging re: reducibility when described as strong (and usually associated with weak emergence).

    Downward causations have also been conceived in strong, medium and weak versions. The strong version, while analogous to the morpho- and teleodynamic processes of biosemiotics and evolutionary biology, refers to a more robustly conceived telos, one that goes beyond science while in no way inconsistent with it. Normatively, it enjoys the epistemic status of our other methodological stipulations and metaphysical presuppositions, which are indispensable to ongoing, meaningful human inquiry, such as various first principles (e.g. noncontradiction, excluded middle, identity) and various common sense notions of causation (e.g. sufficient reason, material, efficient, formal, final, regularities).

     While such methodological stipulations may not be ontologically decisive, no one who takes them as metaphysically suggestive should be regarded as unreasonable, not as long as nontrivial questions regarding primal and/ or ultimate realities seriously beg.

     Those who remain open to the immense heuristic value of Aristotelian causations are making great strides in modeling reality at its various levels of complexity. For example, neuroscientist, Terry Deacon, who studies one end of the chain of being, employing semiotic science, derived ten sign classes from Peirce's nine sign types. This derivation then fostered Sungchul Ji's formulation of a quark model of signs, on the other end of what may one day be considered a pervasively pansemiotic cosmos.

    ReplyDelete
  175. Well, I'm glad that we can join in admiration of Rudy Rucker! :)

    As to causality, I do accept it in general. I think what I'd say is that QM indicates that the Aristotelian account of causality (and maybe all accounts of causality) is incomplete. I also think your earlier points, Dr. Feser, about meanings of terms are worth noting. I think that it's more confusing than helpful in modern times to use the word "cause" for some of Aristotle's four causes, despite the tradition. As I said in the Vox Nova thread, "formal cause" seems to me more like what I'd call "description" or "list of properties". To say, as Aristotle does, that the 2:1 ratio is a formal cause of the octave sounds really weird. I would say that an octave is more an instantiation, or manifestation, or exemplar of the 2:1 ratio. The final cause also sounds odd in that it at least comes off as sounding at times as if nonliving things "want" to do something, or have a purpose in doing something. Since I'm sympathetic to panpsychism, I don't necessarily have a problem with that; but at least in the ordinary way of looking at things, it's odd to say that the "purpose" or "goal" of a falling ojbect is to come to rest.

    Scott, I see what you mean. I'd construe it more as that hydrogen's properties are differently manifested in a molecule rather than that it manifests different properties. As an analogy, a red pigment reflects the red part of the spectrum, and doesn't cease to do so when mixed with yellow pigment (which itself emits the yellow part of the spectrum). However, when red and yellow spectra are received together, we see it is orange. It's not that the behavior of the red pigment changes; it just combines with the behavior of the yellow. Thus, the properties of water are a combination of the interaction of those of hydrogen and oxygen, more than the latter two having "different" properties in a water molecule than when separate. On the other hand, maybe we're just phrasing the same thing in different ways.

    I don't remember the exact book or the exact quote, but Adler seemed to be saying that atoms are virtual in the sense that they actually don't exist unless separated out. I think he said it's like how splinters exist only in potentia in a piece of wood, but need to be--well, splintered off--to exist in actuality. It's been a long time, and I don't remember the exact quote or which book it was in (I'll have to track it down), but I remember having a strong WTF moment upon reading it. What you're saying, though, makes sense.

    ReplyDelete
  176. That doesn't mean that hydrogen just isn't "in" water at all; it's merely a recognition that hydrogen "in" water doesn't manifest the usual properties of, well, hydrogen.

    After all, table salt is a compound of a flammable metal and a poisonous gas! Eek! :-(

    ReplyDelete
  177. @Turmarion:

    "Scott, I see what you mean."

    And I see what you mean as well. I certainly think that at the level of the purely inanimate (the physical and the chemical), there's always some question—which arguably can't be settled empirically—about when a new "substantial form" is introduced or whether the properties of something or other can be regarded as those of a mere aggregate. Does brass, for example, constitute a substance in its own right, or is it just an aggregate of copper and zinc (plus trace elements and impurities)? Should we count a planet as a substance? How about a sedimentary rock? At that level we may indeed be saying the same thing in different words.

    I do think, though, that at higher levels (basically from biology on up) there's a difference. To make the point (or at least a related point) in reverse, Aristotle famously held that a severed human hand isn't properly a "hand" at all: it's a "hand in name only," having been separated from the organic system in which it functions as an organic part. And surely whatever a hand loses by being cut off from a body must be something that it had by virtue of being part of that body.

    ReplyDelete
  178. @TheOFloinn:

    "After all, table salt is a compound of a flammable metal and a poisonous gas! Eek! :-("

    And yet it tastes so good. For some reason that doesn't happen if one ingests the sodium and chlorine separately (or so I've been told; I've never had the courage to try it).

    Yes, that's an excellent example. I'm sure Turmarion will at least see why some essentialists (and I know David Oderberg is on the list) are inclined to regard chemical compounds as having substantial forms of their own.

    ReplyDelete
  179. And just another brief word about this:

    "I'd construe it more as that hydrogen's properties are differently manifested in a molecule rather than that it manifests different properties."

    That's it exactly. For those of us who think that an essence is something over and above properties, and that properties flow or follow from an essence but are not always manifested under all circumstances, it's not easy to distinguish between the two cases (a) hydrogen manifests some (essential) properties under some sets of conditions and others under other sets of conditions, and (b) hydrogen ceases to exist "actually" and begins to exist "virtually" when it's "submerged" (so to speak) in a chemical compound.

    ReplyDelete
  180. Reighley,

    [I]t seems like [Techne has] chosen the least helpful possible notion for the word "random", to the point that nothing random can actually exist. Anyway the issue isn't really about indeterminacy or probability, it is about cause.

    Techne's use of random may not be useful in the sense that "nothing random can actually exist".

    I think this quote, however, implicitly concedes the point Techne's was making, that there is order to the quantum world. Since there is order to the quantum world (it isn't random), it is possible to have causality. If there were no order to quantum world (if it were random), this would imply it is without causality.

    This definition of random is, I think, more useful than having seemingly redundant words (indeterminate and random), and helpful for clarifying what people mean by quantum indeterminacy.

    ReplyDelete
  181. random. There is nothing that requires more careful and intelligent design than to arrange for random outcomes. (See "casinos.") Or how random numbers are actually generated.

    This is a useful essay on the three kinds of scientific investigation:
    http://people.physics.anu.edu.au/~tas110/Teaching/Lectures/L1/Material/WEAVER1947.pdf

    ReplyDelete
  182. Well, Scott, based on your last couple of posts--which are quite good--we seem to be more or less on the same page. We can never quite sort out exactly how it is that forms are instantiated in matter; and in cases like molecular chemistry, it's hard to sort out exactly what a separate "substance" actually is. The edges are really blurry.

    Complex issues like this are why I've said that essentialism vs. nominalism is perhaps something like an "antinomy of pure reason". On the one hand, it doesn't seem possible to give an account of forms that explains all phenomena we observe and that doesn't have at least a certain amount of fuzziness, as we've seen. On the other hand, it seems clearly wrong to deny that universals exist at all. The truth is somewhere in the middle, but it's not something we can ever fully grasp.

    ReplyDelete
    Replies
    1. @Turmarion - Charles Sanders Peirce saw nominalism and essentialism as obverse sides of an epistemic coin with limited purchase on reality. A metaphysical realist, he may have been too harsh on essentialist intuitions because, one way or another, all the moderate realists do seem to be pointing toward the same regularities in nature and affirming common sense notions of causation. It seems to me that a) when Peirce introduced his modal category of thirdness, b) when Scotus introduced his formal distinction and c) even when process-relational thinkers like Hartshorne (improving on rather nominalistic Whiteheadian approaches) refer to such as futurity and generality - those triadic accounts, then, all implicitly affirm something not unlike the Aristotelian notion of potentiality. Beyond the modal categories or states of possibilities/no-thing-ness and actualities, it seems they inescapably must be realists regarding a third mediating category that accounts for
      reality's regularities and tendencies.

      The ontological rub might arise in how we characterize those regularities, for example, whether as necessities or probabilities. Peirce prescinds from metaphysical necessity to probability, where noncontradiction holds but excluded middle folds, hence recognizes what you refer to as fuzziness. Both NC and EM hold, of course, regarding actualities. But his metaphysics are hypothetical and his epistemology fallibilist.

      Peirce does, however, affirm the reasonableness of modal necessity, suggesting it's instantiated in the Ens Necessarium, the reality of God.

      In a Peircean account, we might say that reality's regularities, including its
      morphodynamic influences and formal causes (incl tacit dimensions and boundary conditions) and its teleodynamic influences and final causes (incl mereological and semiotic), are themselves dynamic and emergent, field-like even.

      Distinguished from a nominalism that denies too much or an essentialism that proves too much, most realists, nowadays, are moderate it seems, sufficiently nuanced, whether Thomists, Scotists, Process or such. All of these metaphysical realists seem to converge, whatever their chosen root metaphor. The essentialisms that unrepentant nominalists engage are flimsy caricatures of the highly nuanced scholastic versions?

      Delete
  183. @Turmarion:

    "Well, Scott, based on your last couple of posts--which are quite good--we seem to be more or less on the same page."

    Thank you, and I would say so as well. From your blog posts I suspect we're on at least nearby pages on other issues as well.

    "We can never quite sort out exactly how it is that forms are instantiated in matter; and in cases like molecular chemistry, it's hard to sort out exactly what a separate 'substance' actually is. The edges are really blurry."

    Epistemologically, at any rate. I suppose that if substantial forms are real (as I think they are), then there's some objective matter of fact about whether TheOFloinn's table salt really has a substantial form of its own, but whether that information is available to us is another matter.

    "Complex issues like this are why I've said that essentialism vs. nominalism is perhaps something like an 'antinomy of pure reason'. On the one hand, it doesn't seem possible to give an account of forms that explains all phenomena we observe and that doesn't have at least a certain amount of fuzziness, as we've seen. On the other hand, it seems clearly wrong to deny that universals exist at all. The truth is somewhere in the middle, but it's not something we can ever fully grasp."

    I think I have to raise a possible objection to the opposition of essentialism and nominalism, if it's intended to imply that essences/forms/natures are the only possible candidates for real universals. Some (I'm thinking particularly of Lloyd Gerson) have raised the question whether forms and essences were ever (in Plato's thought) supposed to be "universals" at all, and concluded that they were probably not. Jeffrey Brower has also been kind enough to send me an unpublished paper of his in which he contends that the medieval "problem of universals" was sufficiently different from the modern problem of the same name that Aquinas can be coherently regarded as a realist by one standard and a nominalist by the other; I think he's right, having had similar thoughts myself before asking him about his paper. I suspect that there's simply some more work to be done in sorting out definitions and terminology before the problem can even be adequately addressed.

    ReplyDelete
  184. (For example, the number two and the precise shade of red I see when looking at this apple seem to me to be perfectly viable candidates for "universalhood," but the Scholastics seem to have concentrated generally on what makes two apples both apples, not on whether they "instantiate" a numerically identical twoness or redness. Aquinas's own answer—which as I understand it is that a relation of strictly formal identity obtains between them—seems to me to leave unaddressed the modern problem of whether the two apples can literally share properties.)

    ReplyDelete
  185. I should have written: "[...] it is possible for it to have causality."*

    ReplyDelete
  186. @TheOFloinn,
    "Regarding causation:
    For A to cause B need not mean that precise time and place of B can be predicted. "B is caused"≠"B is predictable." After all, Newton's laws do not predict which apple will fall from which tree and when."

    The notion of prediction introduces all kinds of epistemological baggage that I think misses the point. I imagine myself asking questions of the form "What is the efficient cause of B?". The principle of causality requires that this question have an answer, if it is properly formed. Obviously not all choices of B make the question properly formed. For instance :

    If A is to act on a potency for B then A must be actual and B potential at some point.

    The phrase "at some point" implies a certain comparability in time and space between A and B.

    Now it seems to me that if I put too many limits on B then the principle of causality amounts to "everything that is caused has a cause", and so stops being very interesting. I might even end up in an embarrassing situation in which I could prove the existence of God but not the uniqueness.

    On the other hand if I don't constrain B somewhat further then it seems to me I would be able to fabricate some strange contradictions by asking for the cause of lengths of time, or distances in space. Or by making clever choices of B in order to take measurements of A, and then finding a reference frame in which B came first. Or by making up time shifted objects, as though "B ten minutes from now", is a potency which becomes actual 10 minutes before B itself becomes actual.

    I am not after all concerned with whether the precise time and place of B can be predicted. I am interested in knowing whether "the precise time and place of B" is, like B itself, a thing which has a cause.

    ReplyDelete
  187. @reighley:

    "Now it seems to me that if I put too many limits on B then the principle of causality amounts to 'everything that is caused has a cause', and so stops being very interesting."

    I'm not clear what your objection is here. The Principle of Causality is just that every contingent being has an efficient cause for its coming into being, its remaining in being, and any changes it undergoes—or, more succinctly but with some loss of detail and precision, that whatever begins to exist has an external efficient cause. How does this have anything to do with putting "limits on B," and more generally what does it have to do with predictability?

    "I am interested in knowing whether 'the precise time and place of B' is, like B itself, a thing which has a cause."

    In the example(s) you're addressing, B is an event, not a thing. I would be inclined to say that the efficient causation of "things" is primary and the causation of "events" piggybacks on it. "The precise time and place" of the decay of a certain atom of a lead isotope is in that case just something that depends causally in some way (not necessarily deterministically) on the nature of the atom, which is itself caused by whatever brought it into being in the first place. Where's the problem?

    ReplyDelete
  188. @Turmarion

    http://vox-nova.com/2014/11/26/scholasticism-and-quantum-mechanics-a-question-for-readers/#comment-168867

    “The rock fell because it was in its nature to do so”, i.e. “It fell because it fell,” few would consider that an explanation, let alone a cause."
    - Turmarion

    This, in all demonstrates that you have no understanding of what it means by the formal cause, because any instance of matter is an instance of the form, and any form of matter is an act/potency composite.

    The rock fell because it was in it’s nature do so, seeks to say that it retains an identity through the change, but the change happens, however, this change, the falling of the rock in towards that which it is related with, gravity that is. For instance, a batch of dough cannot become a whale, or a baby whale or a sperm for that matter, before that thing can happen, what needs to take place is a substantial change, where the essence of the thing is totally destroyed, this by no means is tautology, but it is a very simple, and some might call it a trivial point, but it is by no means a claim that is useless.

    This is the explanation if this is something you cannot get or have not understood, then you’ve just not understood the formal cause and you will have to go back to Parmenides and Heraclitus and see what they are arguing before Aristotle comes to resolve the debate.

    “Hard to imagine anyone else could miss the point as badly and consistently as Turmarion does here. Or at least it is until you read LM’s comments. Jeez.” - Edward Feser

    I did not want to agree with this because I thought you were actually interested in the argumentation, but it seems that you are just not interested, this is the basic thing that New Essentialist philosophers have argued, time and again, that matter has different causal powers and dispositions, and cannot do something beyond it’s capability(Act/Potency). The distinction of the formal cause is relevant, because EVERY single instance of matter displays a physical intentionality, and every individuation of matter is limited to it’s causal powers and dispositions, unless of course you believe matter is omnipotent, and also, that it can do what it in essence does not have, this is just some of the absurdities that one has to accept should they deny the formal cause. But then again, I do not know why I answered you here, maybe it’s to simply point to you that you are simply missing the WHOLE point if you think that the formal cause does not give you an adequate understanding of matter works, and how the form relates to it.

    ReplyDelete
  189. @Scott,
    "How does this have anything to do with putting "limits on B," and more generally what does it have to do with predictability?"

    The fact that B has to be a contingent being is a limit on B. The idea of "coming into and remaining in being" and "undergoing change" suggests a time order on the states of B which I don't think all contingent beings necessarily have. So there is another limitation.

    By "a limit on B" I just mean, in the sentence "A caused B", what are allowable substitutions for B? What counts as a contingent being? What are we to make of the relationship of B to the passage of time? It seems like there are physical phenomenon in which these questions are difficult to answer.

    The statement "B is an event, not a thing", seems to imply that events can't be things. Why not? I'm inclined to say that if we are to make any sense of the world at all then such objects as events, facts, statements of facts, intervals of time, logical relationships should all be able to figure in causal chains, both as cause and as effect. If we don't I fear we wouldn't actually be left with very much (matter alone being insufficient).

    On the other hand if I do introduce those things then I have a pretty wicked set of tools with which to pry at (and perhaps even pry apart) the structure of the system.

    I posit that the conceptual difficulties that come with introducing the idea of "causes" into the description of certain physical phenomenon come from giving B (the effects) a very wide latitude and then requiring from A (the causes) a certain logical and physical relationship to B which narrows it so much that, for some B, no suitable A can be found. The idea that B should be predictable from A is just a special case of this.

    The solution is place stricter limits on B, and be more generous with A. Except that if I am too strict with B, I might not be sure that there are any contingent beings at all; And if I am too generous with A I might end up assigning the cause of B incorrectly and having no way, even in principle, of knowing that I had gone wrong.

    It seems to be generally agreed that the statement "Why was the alpha particle brought into being?" is adequately answered by pointing to the radioactive lead and declaring it the cause, but that the statement "Why was the alpha particle not brought into being sooner?" doesn't necessarily have an answer. A time shift and a logical inversion have together totally changed the nature of the question! I just think that's a little disturbing is all.

    ReplyDelete
  190. Turmarion writes...

    "That makes it sound like the seed wants to become a plant, or a boat wants to sail or the ball wants to come to rest at the bottom. “Aim” and “purpose” seem to presuppose a mind or will. I get what Aristotle is saying; but I think the way he frames it is rather confusing."

    No, certainly not, http://edwardfeser.blogspot.com/2009/09/teleology-revisited.html

    I am really sorry to say this, but you have not even understood what it even means by teleology.

    It seems odd to me to say that the ratio of 2:1 causes the octave. I’d say that to say that the 2:1 ratio in frequency exists between two notes an octave apart is more a description than a cause. Aristotle, being a biologist, tends to think of things actualizing their own nature–kind of like a plant or animal–but to me that’s an odd way to look at an inanimate object or a numerical ratio.

    You can only actualize something, if it is *potency*, if it is not in potency, then there is nothing to actualize and it is ALREADY an actual! There is no self-motion required from the rock to fall, in towards something which instantiates gravity. The nature of the thing, is then a certain principle and the cause of stability in the thing, and it is directly present in it. Or, it is IT. In other words, distinction is present within the things that are found in nature, or natural substances. The thing you should be looking at now, is the universals.

    "Being a stone, for example, would be an accidental form of the particles that make up the stone, with (some level of) particles being the entities that really have substantial forms. "

    I don't see what you've said here that disagrees with the formal cause.

    "From this perspective, which is what I hold to, there is no substantial change from “dough” to “bread” to “me” if I bake a loaf of bread and eat it. Rather, the molecules of the dough are rearranged in the rising and baking process, and rearranged again when I digest it and my cells assimilate it."

    It is only because you've misrepresented Feser's position, and taken the quote out of context, an aggregate of parts, presupposes substantial forms at some level of composition, this is not hard to get.

    "....A third point is that the Scholastic metaphysician would hold that the *micro-level parts are in a substance only “virtually” rather than “actually” in the first place, and in a sense depend on the whole as much as the whole depends on the micro-level parts.* Spelling all this out and responding to the usual objections is not something I can do in an already lengthy combox discussion, though -- again, interested readers are referred to my book Scholastic Metaphysics." - Edward Feser


    So, you do not even understand what it means by *substantial change*.....? You've still not understood the formal cause. Please go back to Parmenides and Heraclitus, study what they are arguing about, and then let Aristotle enter. And let me add, you are stil not dealing with the physical intentionality that New Essentialists have been constantly speaking of, let's even grant that you deny it, but you are only reaffirming it by what you write, I don't see how any of what you said goes against the formal cause but only ends up supporting it.

    It's Christmas season, and I'm with Dr. Feser here, please purchase a copy of Scholastic Metaphysics.

    ReplyDelete
  191. http://vox-nova.com/2014/11/26/scholasticism-and-quantum-mechanics-a-question-for-readers/#comment-169003

    Turmarion's post, sorry for the double post myself.

    ReplyDelete
  192. Turmarion: I think that it's more confusing than helpful in modern times to use the word "cause" for some of Aristotle's four causes, despite the tradition. […] To say, as Aristotle does, that the 2:1 ratio is a formal cause of the octave sounds really weird.

    Lots of things sound weird, such as “invisible light”, but it’s just part of the jargon of physics, and one is expected to learn it. It doesn’t sound weird to say that the shape of the teapot’s spout caused condensation to run down a certain way, even though the “shape” is hardly an efficient cause that pushes around droplets of water. Or that the particular composition of the clay in a vase causes it to be brittle. So the jargon is not even that strange on its own, only when applied in contexts where we are used to applying competing concepts; and I have found that by the time one actually understand the concepts well enough, the jargon does not seem problematic any more.

    The final cause also sounds odd in that it at least comes off as sounding at times as if nonliving things "want" to do something, or have a purpose in doing something. […] it's odd to say that the "purpose" or "goal" of a falling ojbect is to come to rest.

    No odder than saying that water “seeks” the lowest point, or that an object “wants” to minimise energy or “wants” to take the shortest path. But again, once one become familiar with the actual concept, as well as the terminology, the reason why certain terms are used makes much more sense.

    I think he said it's like how splinters exist only in potentia in a piece of wood, but need to be--well, splintered off--to exist in actuality. It's been a long time, and I don't remember the exact quote or which book it was in (I'll have to track it down), but I remember having a strong WTF moment upon reading it.

    Sure, because we were all trained to think that everything is “really” just piles of atoms. Aristotle says, no, the stuff is more real than the apparent pile of atoms, not the other way around. But if you can remember that far back, you might recall a similar “whoa” feeling upon being taught that everything is supposedly “really” atoms. It takes some effort to unlearn those things, but it’s worth unlearning things that are wrong.

    ReplyDelete
  193. John West: I had thought quantum objections had more substance to them than they apparently do.

    Not only do QM and Relativity not pose more problems for classical metaphysics, they actually pose fewer than they do for modern views. In fact, I’ve come around to the view that all the “weirdness” in modern physics is greatly exaggerated — the only thing weird about it, really, is that it doesn’t satisfy the prejudices of nineteenth-century physicists. OK, so everything isn’t just tiny billiard-balls; so physicists don’t get to pack it in and go home early, that’s all. There’s nothing freaky or common-sense-defying about physics’ being harder than we hoped it would be over a hundred years ago.

    ReplyDelete
  194. @Edward Feser

    What I termed incoherent was your reference to a series of causes operating simultaneously.

    between series of causes which are essentially ordered and those which are accidentally ordered, and between those which operate simultaneously versus those which are ordered in time.

    If your response is that you were being imprecise in your philosophical discussion, that's fine, but then I am not sure how anything else you derive from this with regards to causality can carry any actual weight (metaphysical or otherwise).

    ReplyDelete
  195. Former World Champion Anatoly Karpov playing 25 chess games simultaneously -- even though he's paying attention to, and/or making a move for, but a single one of the 25 games at any given moment in time.

    ReplyDelete